Anda di halaman 1dari 54

1. Una joven de 14 aos es llevada al mdico por severa dismenorrea desde el pasado ao.

La dismenorrea se acompaa de nusea y vmitos los primeros 2 das de su perodo menstrual y le causa prdida de 2 das de escuela cada mes; la aspirina no le alivia el dolor. Menarquia a los 11 aos de edad. Ella es sexualmente activa con un compaero y usa condom para anticoncepcin. El examen plvico no muestra anormalidades. Los estudios serolgicos muestran una hormona folculo estimulante en una concentracin de 7 mlU/mL, la concentracin de hormona folculo estimulante de 7 mlU/mL, ky la hormona estimulante del tiroides de 4.0 uU/mL. Una prueba de embarazo es negativa. Cul de los siguientes es el paso siguiente mas apropiado en el manejo? A) B) C) D) E) Doxyclina por 10 das Acetaminofn antes de esperada la menstruacin Ibuprofn antes de esperada la menstruacin Codena durante la menstruacin Terapia con Danazol diariamente

Respuesta: C 2. Una enfermera es hospitalizada en el centro mdico donde trabaja por una apendicectoma. Una semana despus de la ciruga, el asistente del administrador del hospital, pregunta al cirujano quien realiz el procedimiento acerca del diagnstico final. Cul de los siguientes es la respuesta ms apropiada de parte del cirujano? A) Respuesta ,por medio del acceso del seguro mdico al centro mdico B) Respuesta, porque como un empleado del centro mdico el administrador tiene acceso a la informacin de los pacientes. C) Respuesta, por la posibilidad de difundir desinformacin acerca del paciente. D) Declina en responder, porque el administrador no es un doctor en medicina. E) Declina en responder, porque la informacin es confidencial. Respuesta: E 3. Un nio de 8 aos con Leucemia linfoblstica aguda ha tenido 3 recadas en los pasados 2 aos. El nico tratamiento disponible es quimioterapia experimental. Sin tratamiento el nio es posible que no sobreviva ms de 6 semanas; con el tratamiento, su pronstico es desconocido. Los padres no quieren ms tratamiento para su hijo y desean llevarlo a casa, el nio tambin desea irse a su casa. Cul de los siguientes es el ms apropiado curso de accin? A) B) C) D) Dar de alta al nio en contra de la orden mdica. Dar de alta al nio como en la forma rutinaria Pedir a la corte una orden de tratamiento Informar a los padres del nio sobre los servicios de proteccin para negligencia mdica.

Respuesta: B

4. Investigadores de la conducta realizan un estudio de cohorte observacional del efecto de la deteccin temprana de pacientes con cncer de pulmn. Los resultados muestran pacientes con cncer del pulmn detectados a travs de un grupo de larga vida, siguiendo el diagnstico, con otros pacientes con cncer de pulmn. Un estudio randomizado separado de cncer de pulmn no demostr ningn aumento en la sobrevida. Cul de los siguientes puntos explica estos resultados dispares? A) B) C) D) E) El estudio randomizado crea confusin para indicacin de tratamiento. En el estudio de cohorte no hay suficiente peso estadstico . Existe sesgo por el tiempo corto en el estudio observacional. Sesgo por el tiempo largo en el estudio randomizado. Misclasificacin de cncer de pulmn en el estudio randomizado.

Respuesta: C 5. Se efecta un estudio para establecer la relacin entre la toxicidad del aluminio y el desarrollo de Enfermedad de demencia tipo Alzheimer. Se escogen 400 pacientes con la enfermedad y 400 controles no afectados, los cuales fueron entrevistados acerca del uso de vajillas de aluminio, tanto de los pacientes como de los controles en la 4. dcada de la vida. Los resultados indican que el relativo uso de vajillas de aluminio es de 2.6 (95% de intervalo de confidencia de 1.9 3.2) Los investigadores concluyen que las vajillas de aluminio causan demencia tipo Alzheimer. Cul de siguientes fallas potenciales es la ms probable para invalidar esta conclusin? A) B) C) D) E) Sesgo en el diagnstico Falacia ecolgica Pobre peso estadstico No es estadsticamente significativo. Sesgo en los pacientes que fueron llamados.

Respuesta : E 6. En un estudio sobre administracin profilctica de Isoniazida, los nios de una escuela , que tenan prueba de tuberculina+, fueron randomizados y asignados para 2 grupos , uno con tratamiento con la droga y otro con placebo. Un tercer grupo, consisti en aquellos no elegidos para entrar en el estudio, que fueron observados. Despus de varios aos, el tercer grupo tuvo una alta prevalencia de tuberculosis comparado con el grupo placebo. Cul es la causa ms probable de estos resultados? A) B) C) D) E) Sesgo del observado. Efecto placebo Pobre randomizacin Sesgo de auto seleccin Sesgo en un estudio no ciego.

Respuesta: D

7. Una mujer de 18 aos, primigrvida con 18 semanas de gestacin se le encuentra antgeno de superficie para hepatitis B+ (HBsAg) en pruebas de rutina. Su esposo fue diagnosticado con hepatitis B, 3 meses atrs. Ella refiere que le intenta dar leche materna a su infante. El examen muestra un hgado firme de 12 cms de largo. Las pruebas sexolgicas para hepatitis muestran: Anti HBsAg IgM-antiHBcAg Anti-HaV negativo positivo negative

Cul de las siguientes opciones teraputicas se debe administrar al recin nacido para prevenir la transmisin de hepatitis? A) B) C) D) Inmunoglobulina de hepatitis B solamente al nacimiento. Vacunacin contra Hepatitis B slo al nacimiento. Inmunoglobulina y vacuna slo al nacimiento. Inmunoglobulina al nacimiento y vacuna con la cesacin de la leche materna. E) Inmunoglobulina de hepatitis B y vacunacin slo si las pruebas sexolgicas hechas al nacimiento para HBsAg y anti HBsAg son negativas. Respuesta: C 8. Un hombre de 55 aos tiene disnea de esfuerzo en los ltimos 10 meses que le impide viajar o subir escaleras. El examen del trax muestra un aumento del dimetro anteroposterior, existe hiperresonancia a la percusin. Los ruidos respiratorias estn distantes y sibilancias al final de la expiracin, respira con dificultad. Cul de las siguientes vacunas debe recibir este paciente anualmente? A) B) C) D) E) Clostridium tetanii Hemophilus influenza tipo B. Virus de la Hepatitis B Virus de la influenza Estreptococo pneumoniae.

Respuesta: D 9. Una mujer de 79 aos se encuentra en un lugar para cuidos de enfermos crnicos; tiene fiebre, cefalea, dolor de garganta, mialgias y tos no productiva. T 39.2C (102.6 F). pulso 104 por min. ,R: 22/ min, P.A. 136/76 mmHg. Apariencia de enferma. Leucocitos 6,200/mm3, N (54%, banda 1%, eos 3%, bas 2%, L 28, M 12%)Una radiografa del trax no muestra anormalidades. Con cul de las siguientes vacunas debe realizarse la inmunizacin? A) B) C) D) BCG Hemophilus influenza tipo b Influenza Pertusis

E) Pneumococo. Respuesta: C virus de la hepatitis B

10. Un nio de 13 aos tiene descarga nasal y prurito de los ojos , cada ao al inicio del verano. Se le practica una prueba de alergia al polen y a los 15 desarrolla eritema de 15 mm en el sitio de la prueba. Cul de los siguientes mecanismos es ms probable para el desarrollo de esta prueba drmica? A) Complejos antgeno / anticuerpo formados en los vasos sanguneos de la piel. B) Instilacin de clulas fagocticas en respuesta a la inyeccin de protenas extraas. C) Liberacin de histamina de los mastocitos. D) Liberacin de linfoquinas por los linfocitos sensibilizados reaccionando con antgenos. E) Liberacin de linfoquinas de los mastocitos. Respuesta: C 11. Varn de 23 aos quien es HIV+, presenta desde hace 2 semanas historia de dolor torcico en mitad del esternn que se agrava con las comidas picantes, el dolor no est relacionado con el ejercicio o posicin, y no refiere disfagia. El tratamiento con bloqueadores H2 no le alivia. Ingiere pastillas de Clotrimazol para candidiasis y Zidovudina (AZT). Tiene CD4+ y cuenta de linfocitos de 220/mm3. Cul de los siguientes pasos es el ms apropiado para su manejo? A) B) C) D) E) Curso teraputico con Acyclovir Investigar pH 24 horas Prueba de perfusin cida Manometra esofgica Esofagoscopa

Respuesta: E 12. Una infante de 9 meses es llevada al mdico a visita de control. Naci a las 33 semanas de gestacin. Su dieta consiste de leche entera y comida de vegetales de baby comercial, frutas y carne. S estatura es la del 50% de su edad y su peso el 75%. El examen no muestra anormalidades a no ser por la palidez. Su hematocrito es de 25%. Un frotis de sangre muestra eritrocitos hipocrmicos, microcticos. Cul de las siguientes opciones es la que mayormente puede prevenir esta condicin? A) B) C) D) E) Suplemento diettico con vitamina C Suplemento diettico con vitamina E Introducir frutas frescas y vegetales a su dieta a los 3 meses de edad. Regular consumo de frmula que contenga hierro. Regular consumo de 2% de leche.

Respuesta: D

13. Una nia africana americana de 3 aos de edad, previamente sana, es llevada al Cuarto de urgencias por epistaxis severa que se controla pobremente al aplicarle presin. Hace 3 meses tuvo otitis media que mejor despus de tratamiento con cloranfenicol oral. Su T es 37C (98F). Est plida y tiene numerosas petequias y equimosis. No tiene linfadenopata o hepatoesplenomegalia. Estudios de laboratorio muestran:

Hb 4.5 g/dL Leucocitos 2000/mm3 con 80% de linfocitos maduros. Reticulocitos 0.2% Plaquetas 12,000/mm3. Cul de los siguientes diagnsticos es el ms probable para explicar estos hallazgos? A) B) C) D) E) Anemia debida a prdida de sangre Anemia aplsica Crisis aplsica de anemia falciforme Mononucleosis infecciosa. Pancitopenia inducida por virus

R: B 14. Un hombre de 40 aos es llevado al Cuarto de Urgencias por alucinaciones desde hace 24 horas; l dej la bebida 3 das atrs. Tiene historia de Diabetes mellitas e Hipertensin. Ingiere medicacin pero no recuerda los nombre de las drogas. Su T es 37.8C (100F), pulso 110/min, P.A. 165/100 mmHg. Est desorientado, tembloroso y diafortico. La medida del hgado es de 16 cm y est sensible a la palpacin. Los laboratorios muestran: Hb: VCM GB: Plaquetas: Suero: Bilirrubina AST 11.2 g/dL 103 Um3 5,000/mm3 50,000/mm3 1.3 mg/dL 120 U/L

Cul de los siguientes puntos es el que explica estos hallazgos? A) B) C) D) E) Interaccin adversa de medicamentos. Hiperesplenismo Mdula mieloptsica Destruccin perifrica de clulas Bloque de produccin de la mdula sea.

Respuesta: E

15. Varn de 54 aos, se presenta con debilidad generalizada desde hace 6 meses, durante este perodo , ha habido una prdida de peso de 9 kg (20 libras). Si T37C (98.6F) pulso 86/min, mientras que posicin de pie 88/min y P.A en decbito supini es de 146/80 mmHg , de pie 142/80 mmHg. El hgado mide 16 cm en la lnea media clavicular derecha. Hb 10 g/dL, Ht 31%, VCM 72 U3, FSP muestra microcitosis e hipocromia. Prueba de sangre oculta es +. Cul de las siguientes causas es la ms probable para explicar estos hallazgos? A) B) C) D) E) F) G) H) I) J) Prdida aguda de sangre Leucemia mieloide aguda Leucemia linfoctica crnica Leucemia mieloide crnica Infeccin por Diphyllobotrium latum Deficiencia enzimtica del eritrocito Deficiencia de cido flico Hemocromatosis Deficiencia de hierro Rasgo de Beta talasemia

R: I 16. Un nio de 6 meses sangra con historia de sangrado fcil, est sangrando por las encas en varias ocasiones desde hace 2 meses. Una ta materna tiene enfermedad hemorrgica. El examen fsico revela pequeas equimosis en las piernas. TPT est prolongado, y el TP es normal. Cul de los siguientes es el factor deficiente ms probable? A) B) C) D) E) Respuesta: C 17. Varn , 18 aos con enfermedad falciforme acude al Cuarto de Urgencias por respiracin corta desde hace 6 horas. Tiene fiebre de bajo grado, dolores articulares, y rash desde hace 3 das. Estuvo previamente bien y ha tenido varias crisis vasoclusivas. Toma cido flico regularmente y no ha requerido transfusiones de sangre por 8 aos. Su Ht ha estado estable en 32%. Su T es 38,1C (1000.6F), pulso 120/min, R 36/min, P.A. 120/86 mmHg. Los pulmones estn claros para percusin y auscultacin. Examen cardaco no muestra hallazgos anormales. No tiene hepato ni esplenomegalia. Tiene dolor con el movimiento de las articulaciones, pero no hay evidencia de artritis. Su Ht es 21%, leucocitos 11,5000 / mm3, cuenta plaquetaria 450,000 /mm3. Una radiografa de trax es normal. Para llegar a un diagnstico, cul es el siguiente paso ms apropiado?. A) Electroforesis de Hemoglobina B) Tiempo de sangra Factor III Factor VII Factor VIII Factor X Factor XIII

C) Cuenta de reticulocitos D) Prueba de HIV E) CAT de abdomen 18. En las ltimas 8 semanas, una mujer de 66 aos con una estenosis severa de la aorta , ha sentido empeoramiento de su respiracin e ingurgitacin venosa yugular. Se auscultan traquidos bilaterales. Se escucha un soplo sistlico, duro, spero, en la punta del corazn, se escucha mejor en la base del corazn. Examen para sangre oculta en el excremento es +. Los estudios de laboratorio muestran: A) Hemoglobina 7.4 g(dL VCM 70 Um3 Cuenta de leucocitos 5400/mm3 Cuenta plaquetaria 580,000/mm3 Eritrosedimentacin 33mm / H Un frotis de sangre perifrica muestra eritrocitos microcticos, hipocrmicos, con moderada poiquilocitosis. Cul es el diagnstico ms probable? A) B) C) D) E) 19. Anemia de la enfermedad crnica Anemia hemoltica autoinmune Anemia por deficiencia de folatos Anemia por deficiencia de hierro Anemia hemoltica microangioptica.

A) Prdida aguda de sangre B) Leucemia linfoblstica aguda C) Leucemia mieloide aguda D) Anemia de la enfermedad crnica E) Anemia aplsica F) Hemlisis autoinmune G) Deficiencia de cido flico H) Deficiencia de hierro I) S-talasemia J) Anemia sideroblstica K) Beta talasemia menor L) Deficiencia de vitamina B12 (cobalamina)

Para cada paciente con anemia, seleccione el diagnstico ms probable: Una mujer de 22 aos con esquizofrenia ha tenido fatiga y disminucin de la energa desde hace 6 meses. Tiene buen apetito, pero ha rehusado comer vegetales frescos por 1 ao porque ha esta oyendo voces que le dicen que los vegetales podran ser venenosos para ella. T 37C (98.6 F), pulso 76 / min, R 18 / min, P.A. 122 /70 mm Hg. Luce plida. El examen fsico y neurolgico es normal. Hb 9.8 g/dL, leucocitos 6,000 /mm3. R: G Un hombre frico americano viene al Cuarto de Urgencias 6 horas despus del inicio de un dolor severo en el trax, y dolor en ambos muslos, los sntomas ocurrieron

mientras estaba esquiando a una altura visitaba Colorado. Tiene dolor obvio. T 38C (104.4 F), pulso 110 /min, , R 24 / min, y P.A. 8000 pies de altura. Vive en New York y ha tenido un episodio previo hace 3 aos, mientras 136 / 84. No tiene dolor a la palpacin de las vrtebras o muslos.No hay anormalidades al examen cardaco o pulmonar. Los estudios de laboratorio muestran: Hemoglobina Leucocitos Reticulocitos R: I 10.5 g/dL 16,000/ mm3 20%

20. Una mujer de 34 aos, previamente en buen estado de salud, es llevada al mdico por fiebre y cefalea de 1 semana. No ha tenido antes ninguna enfermedad. No ingiere medicamentos. T 39.3C (102.8F), pulso 104 / min. Ella est confusa y orientada slo en persona. Al examen fsico muestra ictericia de la piel y conjuntivas. Presenta algunas petequias en el tronco y espalda. No hay linfadenopata. Examen fsico y neurolgico no muestra anormalidades Sangre oculta en heces es +. Los estudios de laboratorio muestran: Hematocrito: y nucleados Leucocitos: Cuenta plaquetaria Tiempo protrombina Tiempo parcial de tromboplastina Productos de degradacin de la fibrina Suero Nitrgeno de urea Creatinina Bilirrubina Total Directa Dehidrogenasa lctica 32% con eritrocitos fragmentados 12,5000/mm3 20,000/mm3 10 30 negativo 35 mg/dL 3.0 mg/dL 3.0 mg/dL 0.5 mg/dL 1000 U/L

Cultivos de sangre y orina son negativos. CAT cerebral no muestra anormalidades. Cul de los siguientes es el diagnstico ms probable? A) B) C) D) E) F) Coagulacin intravascular diseminada Prpura trombocitopnica inmune ;Meningitis meningoccica Sarcoidosis Lupus eritematoso sistmico Prpura trombocitopnica trombtica

Respuesta: F

21. Un varn de 55 aos es llevado a una reseccin quirrgica de urgencia de 40 cm de yeyuno proximal gangrenado debido a una trombosis mesentrica. Fumador de 2 paquetes de cigarrillos diariamente por 35 aos. Laboratorios preoperatorios mostraron: Hematocrito Leucocitos Cuenta plaquetaria Tiempo de protrombina Tiempo de tromboplastina parcial 40% 19,000/mm3 240,000/mm3 12 30 (N=21-36)

Postoperatoriamente, l recibe heparina y antibiticos de amplio espectro. Los estudios al 3er da postoperatorios muestran: Hematocrito Leucocitos Cuenta plaquetaria Tiempo de protrombina Tiempo de tromboplastina parcial 32% 12,000/mm3 30,000/mm3 12.8 65

Cul de los siguientes es el diagnstico ms probable? A) B) C) D) E) Respuesta; C 22. Un varn de 65 aos es llevado al Cuarto de Urgencias por confusin la cual ha ido en aumento en los pasados 4 das. En los ltimos 3 meses ha perdido 9 kg de peso y dolor de espalda progresivamente severo. Toma diurtico tiazdico para hipertensin moderada. Luce letrgico. Su P.A. 156/84. El examen muestra dolor a la palpacin de la espina lumbosacra. Los estudios de laboratorio muestran: Hematocrito Leucocitos Suero Na+ Cloro K+ HCO3Calcio Nitrgeno de urea Creatinina Fsforo Fosfatasa alcalina Acido rico 26% 3200/mm3 144 mEq/L 102 mEq/L 3.7 mEq/L 24 mEq/L 16.8 mg/dL 9.8 mg/dL 5.9 mg/dL 4.0 mg/dL 30 U/L 9.8 mg/dL Insuficiencia adrenal aguda Coagulacin intravascular diseminada Trombocitopenia inducida por heparina Prpura trombocitopnica inmune Prpura trombocitopnica trombtica

Una radiografa de trax no muestra anormalidades. Cul de los siguientes es el diagnstico ms probable? A) B) C) D) E) Respuesta : C 23. A) B) C) D) E) F) G) H) I) J) Leucemia linfoctica crnica Reaccin a droga Enfermedad de Hodgkin Mononucleosis Infecciosa Carcinoma metasttico Sarcoidosis Lupus eritematoso sistmico Toxoplasmosis Tuberculosis Tularemia Carcinoma de pncreas Hipercalcemia inducida por drogas Mieloma mltiple Hipertiroidismo primario Carcinoma de clulas renales.

Para cada paciente con linfadenopata seleccione el diagnstico ms probable. Un varn previamente saludable ha tenido fiebre, sudores nocturnos, prurito y una masa arriba de su clavcula izquierda, desde hace 3 semanas. El examen muestra un ndulo linftico supraclavicular, de 3 cms. no doloroso, cauchoso. Respuesta: C Una mujer de 41 aos acude al mdico para examen de control. Est tomando aspirina por cefaleas crnicas y fenitona por convulsiones desde hace 2 aos. El examen muestra dolor epigstrico moderado a la palpacin y ndulos linfticos no dolorosos de 3 cms en ambas axilas. Una biopsia de uno de los ndulos revela hiperplasia. Respuesta: B 24. un hombre de 68 aos , es hospitalizado para tratamiento de neumona neumoccica. El examen muestra signos de una consolidacin del lbulo inferior derecho del pulmn y ndulos discretos, generalizados, pequeos. La punta del bazo se palpa a 7 cms debajo del reborde costal izquierdo. Los estudios de laboratorio muestran: Hemoglobina Cuenta de leucocitos Cuenta plaquetaria 10.1 g/dL 84,0000/mm3 110,000/mm3

Un frotis de sangre se muestra. Cul de los siguientes es el diagnstico ms probable?

A) B) C) D) E) F) Respuesta: B

Metaplasma mieloide agnognica Leucemia linfoctica crnica Leucemia mieloide crnica Infeccin por el virus de Epstein Barr Enfermedad de Hodgkin Granulocitosis con desviacin a la izquierda

25. Una mujer de 28 aos, acude al Cuarto de Urgencias debido a que 6 hrs. despus del inicio de insomnio, malestar en el flanco, y constipacin que atribuye a clculo renal. Es un residente de otro estado y est visitando a unos parientes. Toma oxycodona (6-8 tabletas diariamente) por dolor crnico de espalda, sumatriptan para migraas , amitriptilina (25 mg en la noche) y paroxetine (30 mg / da) por bulimia nervosa. Pulso 100/min, P.A. 130/80; examen muestra diaforesis y pupilas dilatadas. Est alerta y cooperadora pero luce inquieta. No ha tenido alucinaciones ni ideas suicidas. Se torna enojada cuando se le pregunta por nmeros telefnicos desuss mdicos. La concentracin de amitriptilina srica es 150 mg/dL. Urinlisis muestra 0-2 eritrocitos /hpf. Cul de los siguientes diagnsticos es el ms probable para explicar los sntomas? A) Envenenamiento por anticolinrgicos. B) Deprivacin de opiceos C) Clculo renal D) Sndrome de serotonina E) Intoxicacin tricclica Respuesta: B 26. Un hombre e 18 aos es trado al Cuarto de Urgencias por unos amigos estar tambaleante y sentir pesadez. No tiene antecedentes de problemas mdicos. Tiene aliento alcohlico sin evidencia de trauma. Pulso 70/min, R 6/min, P.A. 110/60. Los pulmones son claros a la auscultacin. S1 y S2. normales. Soplo sistlico de eyeccin. No se palpa el hgado ni el bazo. Pruebas de heces para sangre oculta negativas. Examen neurolgico difcil de realizar. Reflejos+ bilateralmente. Todas las extremidades responden al estmulo doloroso. Su concentracin de alcohol en la sangre es de 200 mg/dL; las pruebas de toxicologa son + para benzodiacepinas en la sangre y embolitos de cocana en la orina. El paciente no responde a la infusin con Naloxone, glucosa o vitamina B1 (tiamina). Cul de las siguientes es la ms probable causa de estos hallazgos? A) B) C) D) E) Solamente alcohol Alcohol y benzodiacepinas Alcohol y cocana Alcohol y feniciclidina Alcohol y salicilatos. B

Respuesta: 27. A) B) C) D) E)

Episodio de amnesia inducido por el alcohol Deprivacin de alcohol Hipertiroidismo apattico Enfermedad bipolar , depresiva Delirio por condicin mdica

F) Demencia relacionada con el alcohol G) Demencia tipo Alzheimer H) Desorden de ansiedad generalizado I) Depresin enmascarada J) Toxicidad medicamentosa K) Prdida de la memoria asociada a la edad L) Hidrocfalos con presin normal M) Enfermedad de Parkinson N) Enfermedad de Pick O) Pseudodemencia P) Esquizofrenia residual Q) Demencia por multi-infartos (vascular) Para cada paciente con un problema de memoria, seleccione el diagnstico ms probable. Una mujer de 29 aos , con una historia de enfermedad bipolar de 11 aos , acude al mdico por prdida de la memora en las ltimas 2 semanas. Ha tenido dificultad en recordar citas que ella ha hecho, y en una ocasin, lleg a olvidar el club de salud en el cual ha sido miembro por aos. Ha estado tomando carbonato de litio por 8 aos, y ha estado tomando un diurtico tiazdico por aumento de peso perimenstrual en los pasados 3 meses. El examen fsico muestra un temblor de reposo en ambas manos y ataxia moderada. El examen mental demuestra que est orientada en persona, lugar y tiempo pero retiene solamente 1 objeto de 3, en 5 minutos. Respuesta: J toxicidad al carbonato de litio

Un hombre de 63 aos es llevado al mdico por su hija debido a prdida de la memoria en el pasado ao. Ayer no pudo recordar el nombre de su nieto de 18 meses . Aunque niega que exista algn problema, ella dice que l se ha vuelto olvidadizo y se torna fcilmente confuso. No hay historia de abuso de alcohol. Su temperatura es 37C (98.6F), pulso 77/min, respiraciones son 12 / min y P.A.118/84 mmHg. Al examen mental su humor es normal. Est orientado en persona y lugar, pero inicialmente da el mes errado, el cual es capaz de corregir. Retiene la memoria desde su juventud en gran detalle, pero slo retiene una de 3 palabras despus de 5 min. Tiene dificultad reteniendo nombre de objetos comunes y no recuerda e nombre del actual Presidente de los Estados Unidos. El examen fsico , laboratorios y pruebas de funcin tiroidea no muestran anormalidades. Respuesta: G demencia tipo Alzheimer

28. Una adolescente de 17 aos previamente saludable es llevado al mdico por prdida del apetito, insomnio y por irritabilidad extrema desde hace 3 semanas. Se siente con frecuencia cansada y tiene dificultad para permanecer sentada y concentrarse en su trabajo escolar. Su menstruacin ocurre a intervalos regulares. Mide 168 cm (56) y pesa 50 kg (110 lbs); su ndice de masa corporal (BMI) 18 kg/m2. Pulso 74 / min, R 16 / min y P.A. 110 /70 mmHg. Cul es el diagnstico ms probable? A) Reaccin de ajuste con disturbio mixto emocional y de conducta

B) C) D) E) Respuesta: E

Anorexia nervosa Dficit de atencin con hiperactividad Enfermedad distmica Enfermedad depresiva mayor

29. Una mujer de 54 aos acude al mdico 2 meses despus de que su esposo muri de infarto del miocardio. Se cansa fcilmente y tiene dolor torcico cuando inhala profundamente en reposo. Ha tenido frecuentes llantos cuando habla, asociados con pensamientos con su esposo y siente que no se siente comprometida con sus actividades diarias. Cual es el diagnstico ms probable? A) B) C) D) E) Respuesta: D 30. Una mujer de 45 aos tiene historia de 2 semanas de ansiedad, dolor abdominal, irritabilidad, y dificultad en la concentracin; a ella le robaron con un cuchillo en un estacionamiento, hace 3 semanas. Toma Levotirosina para hipotiroidismo y usa un inhalador para asma. P.A. 140/80 mmHg, y pulso 100/min. El examen muestra pie y cabello secos. Es cooperadora, pero luce ansiosa, y se sobresalta cuando un ruido se escucha fuera de la oficina. Leucocitos 12,000/mm3, TSH 5.0 uU/mL. Un ECG muestra taquicardia sinusal Cul de los siguientes es el diagnstico ms probable? A) B) C) D) E) Estrs agudo Agorafobia Desorden de ansiedad generalizada Hipotiroidismo Desorden de pnico Desorden de conversin Desorden de ansiedad generalizada Desorden depresivo mayor Reaccin de duelo o pena normal Reaccin de duelo o pena patolgica.

Respuesta: A

Para cada paciente con ansiedad, seleccione el diagnstico ms probable. A. B. C. D. E. F. Sndrome de Abstinencia Alcohlica Insuficiencia Coronaria Hipertiroidismo Hipoglicemia Trastorno Depresivo Mayor Trastorno de Pnico G. H. I. J. K. Taquicardia Paroxstica Atrial Feocromocitoma Estrs Postraumtico Embolismo Pulmonar Trastorno Somatoforme

1. Una mujer de 33 aos acude al cuarto de urgencias 30 minutos despus por presentar dolor torcico sbito, palpitaciones, respiracin entrecortada, adormecimiento y punzadas en ambos brazos y miedo a volverse loca. Ha visitado el cuarto de urgencias local varias veces los ltimos 3 meses por presentar sntomas similares que resuelven a la hora. Utiliza anticonceptivos orales. Bebe dos cervezas diariamente y seis cervezas los fines de

semana. No tiene historia de problemas mdicos. Su madre y hermanas tienen historia de ansiedad. Su pulso es de 90 por minuto, su respiracin de 18 por minuto, su presin arterial de 130/90 mmHg. El examen fsico, los estudios de laboratorio y el electrocardiograma no muestra alteraciones. Cul es el diagnstico ms probable? Respuesta F (Trastorno de Pnico) 2. Una mujer de 35 aos acude al mdico por nerviosismo, temblores, labilidad emocional y excesiva sudoracin desde hace 3 semanas; ha tenido una perdida de peso de 4.5Kg (10lbs) durante este periodo. No tiene antecedentes personales o familiares de enfermedades psiquitricas. Su pulso es de 95 por minuto, con 12 respiraciones por minuto y una presin sangunea de 120/80 mmHg. El examen muestra piel caliente y hmeda, un fino temblor en sus dedos y lengua e hiperreflexia. Cul es el diagnstico ms probable? Respuesta C (Hipertiroidismo)

3. Un nio de 10 aos es trado al mdico por haber incrementado sus problemas de comportamiento en la escuela desde el comienzo del quinto grado hace 3 meses. Su maestra refiere que el no puede permanecer sentado quieto durante el periodo de clases y frecuentemente interrumpe la clase y a otros nios mientras estn hablando. Sus padres reportan que l ha sido siempre un nio activo y se han interesado porque es inatento cuando camina o corre. Durante el examen, tiene el afn de mover constantemente sus manos y pies y es fcilmente distrado para completar una tarea. Cul de los siguientes es la ms apropiada farmacoterapia? A. B. C. D. E. Amitriptilina Fluoxetina Haloperidol Imipramina Metilfenidato

Respuesta E Diagnstico Dficit Atencional/Trastorno de Hiperactividad 4. Un infante de 3 meses de edad es trado al mdico por sus padres porque contina teniendo fiebre, dolor de odo e irritabilidad despus del tratamiento con amoxicilina por 72 horas. Al examen se evidencia una membrana timpnica inmvil, roja y opaca. Cul de los siguientes es el agente etiolgico ms probable? A. B. C. D. E. Chlamydia trachomatis Estreptococo del Grupo A Haemophilus influenzae Mycoplasma pneumoniae Staphylococcus aureus Diagnstico Otitis Media

Respuesta C

5. Un estudiante de 19 aos es trado al cuarto de urgencias por su compaero de cuarto por presentar dificultad para despertarlo en la maana. Ha tenido algo parecido a la gripe con fiebre, dolores musculares y articulares desde hace 12 horas. Su temperatura es 39.5C (103.1F), pulso de 120 por minuto, una presin arterial de 90/60 mmHg. Hay un rash petequial difuso sobre el tronco y las extremidades. Tiene rigidez nucal que no puede flexionar pasivamente. Cul de los siguientes es el agente etiolgico ms probable? A. B. C. D. E. Coxsackievirus B Echovirus Haemophilus influenzae Neisseria meningitidis Streptococcus pneumoniae Diagnstico Meningitis

Respuesta D

6. Cerca de la semana pasada, una mujer de 55 aos ha tenido vrtigo extremo al acostarse o levantarse. Refiere que se siente como si el cuarto diera vueltas. El examen no muestra alteraciones excepto por un nistagmo fatigable hacia el lado izquierdo cuando la paciente se deja caer sobre su lado izquierdo. Cul de los siguientes es el diagnstico ms probable? A. B. C. D. E. Vrtigo Posicional Benigno Enfermedad de Meniere Fstula Perilinftica Tumor del glomus yugular Neuronitis Vestibular

Respuesta A Diagnstico Vrtigo Posicional Benigno Para cada paciente con ansiedad, seleccione el diagnstico ms probable. A. B. C. D. E. F. Sinusitis Aguda Sinusitis Crnica Cefalea en Racimos Tumor Intracraneal Meningoencefalitis Migraa G. H. I. J. K. Feocromocitoma Cefalea Postraumtica Hemorragia Subaracnoidea Arteritis Temporal Sndrome Temporomandibular

7. Un hombre de 43 aos acude al cuarto de urgencias a las 3:00am por presentar una cefalea severa constante desde hace 1 hora. El dolor es localizado detrs de los ojos. Tambin ha tenido secrecin acuosa por la narina izquierda. Ha tenido episodios similares nocturnos desde de la semana pasada. Tiene historia de sntomas similares en periodos de cada 3 semanas por los ltimos 2 aos. Su pupila izquierda es ms pequea

en comparacin con la derecha y hay una ptosis palpebral en el lado izquierdo. Se observa una secrecin hialina en la narina izquierda. Cul es el diagnstico ms probable? Respuesta C (Cefalea en Racimos) 8. Una mujer de 19 aos acude al mdico por presentar un cuadro de cefaleas recurrentes desde hace 1 ao. Las cefaleas son unilaterales y pulstiles acompaadas de nuseas, vmitos y fotosensibilidad. Las cefaleas ocurren una a dos veces al mes alcanzando su mxima intensidad a la hora con una duracin de 12 a 24 horas. No hay aura. El examen no muestra anormalidades. Cul es el diagnstico ms probable? Respuesta F (Migraa) 9. Una mujer de 28 aos de edad tiene palpitaciones que ocurren aproximadamente desde hace 1 semana, duran aproximadamente de 1 a 5 minutos y consisten de un latido rpido y regular. Los episodios inician y terminan sbitamente y no son asociados con molestias torcicas o disnea. No hay historia de problemas cardacos. Toma 2 a 3 tazas de caf diariamente. Raramente toma alcohol y no fuma. Su pulso es de 96 por minuto y regular, su presin arterial es de 120/88 mmHg. Al examen se ha notado exoftalmos, la glndula tiroidea es firme y 1.5 veces su tamao normal. Hay un clic medio sistlico en la punta y un soplo sistlico 2/6 en el borde esternal izquierdo. El electrocardiograma es normal excepto por taquicardia sinusal. Cul de los siguientes es el paso ms apropiado en el diagnstico? A. B. C. D. E. Monitoreo Electrocardiogrfico Ambulatorio Medicin Srica de la Hormona Estimulante del Tiroides Medicin de la Concentracin de Catecolaminas Centelleo MUGA Ecocardiograma

Respuesta B Diagnstico Enfermedad de Graves Para cada paciente con ansiedad, seleccione el diagnstico ms probable. A. B. C. D. E. F. Polineuropata Diabtica Enfermedad de Huntington Sndrome Medular Lateral Encefalopata por Plomo Meduloblastoma Esclerosis Mltiple G. H. I. J. Enfermedad de Parkinson Glioma Pontino Tabes dorsalis Encefalopata de Wernicke

10. Un hombre de 50 aos es llevado al mdico por su esposa por presentar historia de 1 mes de incremento gradual de la confusin mental, desorientacin y perdida de la memoria a corto plazo. El examen no muestra anormalidades excepto por dejar caer su pie izquierdo. Un frotis de sangre perifrica muestra microcitosis y eritrocitos con moteado basoflico. Cul es el diagnstico ms probable? Respuesta D (Encefalopata por Plomo) 11. Una joven de 20 aos es llevada al cuarto de urgencias 30 minutos despus de haber presentado visin doble en forma aguda en su ojo izquierdo. Hace 2 meses, ha tenido un episodio agudo de dolor al movimiento del ojo izquierdo con perdida de la visin tanto central como del reflejo pupilar a la luz; estos sntomas mejoraron espontneamente. El

examen muestra un retraso en la adduccin del ojo izquierdo y nistagmo en el ojo derecho al voltear hacia el lado derecho. Cul es el diagnstico ms probable? Respuesta F (Esclerosis Mltiple) 12. Un hombre de 57 aos acude al cuarto de urgencias por presentar un esputo sanguinolento desde hace 2 semanas. Tiene una historia de 6 meses presentando disnea excepcional especialmente cuando camina cuesta arriba o sube las escaleras. No toma ninguna medicacin y no fuma. Es alrgico a la penicilina. Su pulso es de 88 por minuto, respiraciones de 16 por minuto, con presin arterial de 120/80 mmHg. Se auscultan crpitos en ambas bases pulmonares y un soplo diastlico puede ser auscultado en el pice cardaco. El electrocardiograma muestra una ancha muesca de la onda P. La radiografa de trax muestra redistribucin de los vasos pulmonares hacia los lbulos superiores. Cul de los siguientes es el paso ms apropiado en el diagnstico? A. B. C. D. E. Broncoscopa Angiografa Coronaria Angiografa Pulmonar Ecocardiograma Cateterizacin de la Arteria Pulmonar

Respuesta D Diagnstico Estenosis Mitral 13. Un joven de 15 aos es trado al cuarto de urgencias una hora despus de haber presentado un episodio de sncope mientras corra en una carrera de 400 metros. Ha tenido episodios similares hace 2 aos. Su madre y primo hermano materno fallecieron sbitamente a las edades de 32 y 17 aos, respectivamente. El examen muestra abrasin de su cara, manos y rodillas. El examen cardiolgico y neurolgico no evidencia anormalidades. Cul de los siguientes es el paso ms apropiado en el diagnstico? A. B. C. D. E. Prueba de Inclinacin Tomografa Cerebral Electrocardiograma Electroencefalograma Puncin Lumbar Diagnstico Sndrome del Intervalo QT Prolongado

Respuesta C

14. Un nio de 10 aos con sinusitis crnica es trado al cuarto de urgencias seguido de 3 minutos de convulsiones tnico-clnicas generalizadas. Tiene una historia de 3 das de incremento severo de cefaleas y 2 das de vmitos y temperatura de 38.7C (101.7F), pulso de 80 por minuto y respiraciones de 16 por minuto con presin arterial de 135/90 mmHg. Est ligeramente letrgico pero responde a las preguntas apropiadamente. El examen muestra papiledema. No se evidencian otras anomalas. Cul de los siguientes es el paso ms apropiado en el manejo? A. Medicin de la Concentracin Srica de Amonio B. Radiografa de Seno Paranasales C. Electroencefalograma

D. Tomografa Cerebral E. Puncin Lumbar Respuesta D Diagnstico Absceso Cerebral

15. Un hombre de 32 aos ha tenido debilidad progresiva en sus brazos y piernas los pasados 4 das. Se ha sentido bien excepto por una infeccin del tracto respiratorio superior hace 10 das. Su temperatura es 37.8C (100F), pulso de 94 por minuto, respiracin superficial de 42 por minuto y presin arterial de 130/80 mmHg. Hay una debilidad simtrica en ambos lados de su cara y de los msculos proximales y distales de las extremidades tanto superiores como inferiores. La sensibilidad est intacta. No hay reflejos tendinosos profundos; la respuesta plantar es flexora. Cul de los siguientes es el diagnstico ms probable? A. B. C. D. E. Encefalomielitis Aguda Diseminada Sndrome de Guillain Barr Miastenia Gravis Poliomielitis Polimiositis

Respuesta B Diagnstico Sndrome de Guillain Barr Para cada paciente con ansiedad, seleccione el diagnstico ms probable. A. B. C. D. E. F. Pericarditis Aguda Shock Cardiognico Pericardiopata Constrictiva Crnica Enfermedad de la Vlvula Mitral Miocarditis Taponamiento Pericardiaco G. H. I. J. Pleuritis Embolismo Pulmonar Fiebre Reumtica Neumotrax Espontneo

16. Un joven de 18 aos previamente saludable acude al cuarto de urgencias 12 horas despus de presentar dolor torcico en el rea del msculo trapecio. Ha tenido una infeccin del tracto respiratorio superior por 9 das. Se auscultan crpitos sobre el precordio. Un electrocardiograma muestra incremento del punto J en todas las derivadas con excepcin de aVR y V1. Despus de la administracin de aspirina, el dolor cede. Cul es el diagnstico ms probable? Respuesta A (Pericarditis Aguda) 17. Una mujer de 42 aos acude al cuarto de urgencias 1 hora despus de presentar sbitamente dolor torcico, tos, disnea, taquipnea y marcada ansiedad. Hace 2 das, se le realiz una hemicolectoma por cncer en el colon ascendente. Se ausculta un acentuado ruido S2 en la pulmonar. El electrocardiograma muestra cambios no especficos del segmento ST y de la onda T. El conteo leucocitario es de 12000/mm3. La radiografa de trax no muestra infiltrados pulmonares ni efusin pleural. La gasometra arterial al aire ambiente muestra una PCO2 en 30 mmHg y una PO2 en 55 mmHg. Cul es el diagnstico ms probable? Respuesta H (Embolismo Pulmonar)

18. Un hombre de 58 aos acude al mdico por presentar fatiga extrema e indisposicin por 3 semanas. Se ha sentido bien excepto por un dolor de muelas hace 5 semanas manejado con un procedimiento en la raz del canal. Tiene una historia de un soplo cardaco notado la primera vez a la edad de 19 aos. Su temperatura es de 37.8C (100F), pulso de 110 por minuto y presin arterial de 120/80 mmHg. Los pulmones se auscultan claros. El examen cardaco muestra un soplo sistlico de eyeccin 2/6 escuchado mejor en el segundo espacio intercostal derecho tambin como un S4 y un clic de eyeccin. Los laboratorios muestran: Hemoglobina 9.3h/dl Leucocitos 10000/mm3 Veloc. de Sedimentacin 90mm/h Neutrofilos Segmentados 90% Hematuria Positivo Bandas 10%

El Hemocultivo es obtenido. Cul tincin de Gram del Hemocultivo es el ms probable? A. B. C. D. E. Cocos en Cadenas Gram Positivos Cocos en Racimos Gram Positivos Diplococos Gram Positivos Bacilos en cadenas Gram Negativos Diplococos Gram Negativos

Respuesta A Diagnstico Endocarditis Infecciosa 19. Una joven de 14 aos moderadamente obesa tiene historia de 2 semanas presentando cefaleas bifrontal severa y vmitos temprano en la maana. Aparece alerta y cooperadora. Ella es diestra. Su pulso es de 82 por minuto y la presin arterial es 112/76 mmHg. El fondo de ojo evidencia lo de la figura. Su agudeza visual es de 20/20 bilateralmente; el examen neurolgico y el CAT cerebral no evidencia anormalidades. Cul de los siguientes es el diagnstico ms probable? A. B. C. D. E. Migraa Neuritis ptica Tumor de Fosa Posterior Pseudotumor cerebri Cefaleas Tensional Diagnstico Pseudotumor cerebri

Respuesta D

20. Un joven de 18 aos jugador de bsquetbol en la secundaria acude al mdico para un examen fsico de rutina. Mide 193cm (6 pies 4 pulgadas); su brazada mide 201cm (79 pulgadas). Tiene largos dedos en las manos y pies. Su pulso es 64 por minuto, su presin arteria es 146/62 mmHg. Presenta un soplo diastlico de alta frecuencia 2/6 en el segundo espacio intercostal derecho. Cul de los siguientes es el diagnstico ms probable? A. B. C. D. E. Regurgitacin Artica Defecto Septal Atrial Coartacin de la Aorta Anomala Ebstein Estenosis Mitral

Respuesta A

Diagnstico

Sndrome de Marfan

21. Una mujer de 54 aos es trada al cuarto de urgencias por sus familiares por presentar adormecimiento y parlisis de sus extremidades superior e inferior izquierdos desde hace 2 horas. Tiene historia de hipertensin y abuso de alcohol y ha fumado 2 paquetes de cigarrillos diarios desde hace 30 aos. La sensacin est disminuida en la extremidad superior e inferior izquierdo y la fuerza muscular es 0/5. Se escucha un soplo carotdeo derecho. Cul de los siguientes es el estudio inicial ms apropiado? A. B. C. D. E. Electroencefalograma Doppler Carotdeo Tomografa Cerebral Ecocardiograma Angiografa Carotdea

Respuesta C Diagnstico Infarto Cerebral Para cada paciente con ansiedad, seleccione el diagnstico ms probable. A. B. C. D. E. F. G. H. I. J. K. L. Angina pectoris Diseccin Artica Estenosis Artica Artritis de la Espina Enfermedad Disco Cervical Pericardiopata Crnica Constrictiva Costocondritis Aneurisma Disecante de la Aorta lcera Duodenal Espasmo Esofgico Sndrome de Atrapamiento de Aire Herpes zoster M. Hernia Hiatal N. Cardiomiopata Hipertrfica O. Cncer Pulmonar P. Enfermedad Metastsica Pulmonar Q. Vlvula Mitral Prolapsada R. Infarto al Miocardio S. Miocarditis T. Taponamiento Cardaco U. Neumona V. Neumotrax W. Embolismo Pulmonar

22. Un hombre de 37 aos acude al cuarto de urgencias 1 hora despus de presentar un dolor torcico anterior. El dolor se irradia a la espalda y brazo izquierdo. Mide 203cm (6 pies 7 pulgadas) y pesa 86Kg (190lbs); IMC es 21Kg/m2. Su temperatura es 36.7C (98F), pulso de 116 por minuto y respiracin de 20 por minuto, con presin arterial en 115/70 mmHg. Los pulmones se auscultan claros. El electrocardiograma es normal. Cul es el diagnstico ms probable? Respuesta B (Diseccin Artica) 23. Un hombre de 72 aos acude al cuarto de urgencias 2 horas despus de haber presentado sbitamente dolor torcico anterior y respiracin entrecortada. El dolor es exacerbado por la respiracin profunda. Le fue diagnosticado cncer prosttico metastsico hace 6 meses atrs. Su temperatura es 37.2C (98.9F), pulso de 136 por minuto, respiracin de 28 por minuto y la presin arterial es 100/56 mmHg. A la auscultacin cardiopulmonar no se encuentran anormalidades. El electrocardiograma muestra una desviacin hacia el axis derecho que no se evidenciaba en el electrocardiograma de hace 1 mes atrs. Cul es el diagnstico ms probable? Respuesta W (Embolismo Pulmonar)

24. Un recin nacido de 2 semanas es trado al mdico porque sus labios se han tornado azules en tres ocasiones durante la alimentacin; adems suda mucho durante la alimentacin. Naci a las 38 semanas de gestacin y peso 2466g (5lbs 7oz); corrientemente pesa 2778g (6lbs 2oz). Su temperatura es 37.8C (100F), pulso de 170 por minuto, respiracin de 44 por minuto y presin arterial de 75/45 mmHg. Se ausculta un soplo rudo sistlico de eyeccin 3/6 en el borde esternal superior izquierdo. La radiografa de trax evidencia una pequea figura de botn en el corazn y disminucin de la trama vascular pulmonar. Cul de los siguientes es el diagnstico ms probable? A. B. C. D. E. Vasos Coronarios Anmalos Defecto Atrioseptal Fibroelastosis Endocardial Tetraloga de Fallot Anomala Total del Retorno Venosos Pulmonar

Respuesta D Diagnstico Tetraloga de Fallot 25. Una mujer de 64 aos de edad viene al mdico por historia de 5 meses de evolucin de incremento en dificultad respiratoria, dolor de garganta y tos productiva con esputo blanco escaso. La pasada semana, ella haba tenido nuseas relacionada a tos excesiva. En el ao pasado, ella ha perdido 7 lbs. Ella tiene asma tratada con teofilina y agonistas B adrenrgicos y corticoides. Ella ha fumado un paquete de cigarrillos diario por 44 aos y toma una bebida alcohlica diaria. Se observa delgada. El examen muestra un ndulo de 2cm no doloroso en el rea supraclavicular derecha. El examen no muestra otras anormalidades. Una radiografa de trax muestra una gran densidad en lbulo inferior derecho. Un CAT de trax muestra una masa en lbulo inferior derecho de 7.5 x 7.5 x 6cm con algunas calcificaciones especuladas. La lesin se acerca a la pared posterior del trax sin una clara invasin. Hay ndulos linfticos peritraqueales derechos, precarinales, hiliares derechos y subcarinales. Hay una masa de 1.5cm en la glndula adrenal derecha. Una biopsia de la masa pulmonar demuestra lo siguiente: A. B. C. D. E. F. G. H. Linfoma de Clulas B Absceso Pulmonar Aesotelioma Adenocarcinoma de Mama Metastsico Neoplasia Endocrina Mltiple Carcinoma de Pulmn de Clulas No Pequeas Sarcoidosis Tuberculosis Diagnstico Cncer de Pulmn

Respuesta F

26. Dos horas despus de una colecistectoma sin eventualidades, una mujer de 50 aos de edad tuvo extrasstoles y una disminucin en la presin sistlica de 110 a 90 mmHg. Los gases arteriales a aire ambiente muestran: pH 7.30, Pco2 52 mmHg, pO2 58 mmHg Cul de los siguientes es el diagnstico ms probable?

A. B. C. D. E.

Hipoventilacin Alveolar Hemorragia Oculta Fallo e irritabilidad Cardaca Hipoxemia Primaria Causada por Anestsicos Embolismo Pulmonar Diagnstico Hipoventilacin Alveolar.

Respuesta A

27. Dos das despus de una fractura femoral, un hombre de 25 aos de edad llega confundido y disnico. El examen muestra petequias sobre el trax. Su temperatura es 38.2C, pulso 115 por minuto, y presin sangunea 110/70 mmHg. Cul de los siguientes es la causa ms probable de estos hallazgos? A. B. C. D. E. Reaccin adversa a Analgsicos CID (Coagulacin Intravascular Diseminada) Embolia Grasa Shock Hipovolmico Trombocitopenia

Respuesta C Diagnstico Embolia Grasa 28. Un hombre de 21 aos de edad es trado al cuarto de urgencias 45 minutos despus de una colisin frontal en vehculo de motor, en la cual el era el conductor. A su llegada, el est alerta y coherente, respirando oxgeno 100. El examen muestra equimosis en la porcin central del trax extendido a ambos lados, las venas del cuello no estn distendidas, su trquea est desviada a la derecha. Los ruidos respiratorios estn presentes en el lado derecho y ausentes en el izquierdo, luego de la administracin de 2 litros de Lactato Ringer, su presin sistlica es 80 mmHg. Cul de los siguientes es el ms apropiada prximo paso en el manejo? A. B. C. D. E. Intubacin Insercin de Tubo de Toracostoma Izquierda Infusin de Sangre No Cruzada. Lavado Peritoneal Pericardiocentesis Diagnstico Hemotrax

Respuesta B

29. Un hombre de 19 aos de edad ha tenido malestar general, dolor de garganta recurrente, anorexia, edema de glndulas, y prdida de peso de 9 lbs en las ltimas 5 semanas, un cultivo de garganta creci Streptococcus pyogenes (grupo A), y el paciente fue tratado con penicilina. Su temperatura es de 38.5C. El examen demuestra faringitis exudativa, linfadenopata anterior y cervical posterior, y esplenomegalia. Los laboratorios demuestran: Hcto 37 %, GB 3200/mm3, Neutrfilos 55%, Linfocitos 34 %, Plaquetas 84,000/mm3, AST srica 75 u/L. Cul de los siguientes es el paso diagnstico ms apropiado? A. Estudios de Anticuerpos Antiplaquetarios B. Estudios de Antiestreptolisina O C. Examen de Mdula sea

D. Tamizaje por Hepatitis E. Serologa por Ebstein Barr Virus Respuesta B Diagnstico Infeccin por Ebstein Barr virus

30. Un hombre de 22 aos de edad con un desorden convulsivo, ha tenido un incremento en tos y respiracin corta, por 3 das y fiebre por 1 da. El tiene esputo maloliente. El tuvo una convulsin tnico clnico generalizada una semana antes. Su temperatura es 39.4C. Se auscultan crpitos en trax. Una radiografa de trax muestra un infiltrado en lbulo superior derecho. Cul de los siguientes es el diagnstico ms probable? A. B. C. D. E. Neumonitis Qumica Neumona por Anaerobios Neumona Secundaria a Aerobios Gram Negativos Neumona Secundaria a Aerobios Gram Positivos Neumona Secundaria a Micoplasma pneumoniae

Respuesta B Diagnstico Neumona por Aspiracin Para cada paciente de los siguientes con tos, selecciones el Diagnstico ms probable: A. Bronquitis Aguda B. Asma C. EPOC D. Falla Cardaca Congestiva E. Fibrosis Qustica F. Cncer Metastsico a Pulmn G. Neumona H. Carcinoma Primario de Pulmn I.Hipertensin Pulmonar Primaria J. Embolismo Pulmonar K. Sarcoidosis L. Apnea del Sueo M. Tuberculosis N. Pleuritis Viral

31. Un hombre de 59 aos de edad ha tenido tos y esputo productivo con estras de sangre por 8 horas. Su temperatura 38.9C, malestar, respiracin corta, tos productiva de esputo amarillo por 2 das. El ha fumado 2 paquetes diarios por 40 aos. Roncus, sibilancias y egofona se escuchan en el hemotrax derecho. Una radiografa de trax muestra un infiltrado lobar denso en hemotrax derecho. Cul es el diagnstico ms probable? Respuesta G (Neumona) 32. Un hombre de 25 aos de edad ha tenido tos productiva con esputo con estras de sangre por 5 das. En los pasados 6 aos, el ha tenido sntomas similares una o dos veces por ao, que han sido tratadas satisfactoriamente con antibiticoterapia. el ha tenido tos por 12 aos, recientemente productiva de una a dos tazas de esputo verde amarillo diarias. No fuma, toma lipasa, proteasa y amilasa por mala absorcin. Con roncus bilaterales y sibilancias. Una radiografa muestra incrementote marcas intersticiales bilaterales, lesiones nodulares e hiperinflacin. Cul es el diagnstico ms probable? Respuesta E (Fibrosis Qustica) 33. una mujer de 47 aos de edad ha tenido tos productiva de escasa cantidad de esputo amarillo, su temperatura de 38.3C por 3 meses. En una ocasin, el esputo fue con estras de sangre. Ha perdido 21 lbs de peso, en los ltimos 4 meses, ella no fuma. Es inmigrante de Vietnam hace 20 aos su registro mdico no est disponible. Se ausculta crpitos en lbulo superior y roncus bilaterales. Una radiografa mostr infiltrados mltiples, bilaterales y cavitaciones en lbulo superior rodeados de infiltrados pulmonares. Cul es el diagnstico ms probable? Respuesta N (Tuberculosis)

34. Un hombre de 50 aos de edad con hipertensin es trado al cuarto de urgencias 30 minutos despus del inicio sbito de un dolor torcico que se irradia a espalda y brazos. Su presin sangunea es 180/80 en su brazo izquierdo no se puede obtener PA en su brazo derecho. Al examen cardaco se ausculta un soplo de insuficiencia artica. Cul de los siguientes es el diagnstico ms probable? A. Diseccin Artica Aguda B. Infarto Agudo del Miocardio C. Embolismo de la Arteria Subclavia Derecha D. Embolismo Pulmonar E. Neumotrax Espontneo Respuesta A Diagnstico Diseccin Artica Aguda 35. Un hombre de 24 aos de edad operador de un jackhammer, viene al mdico por historia de una semana de dolor, edema que afecta todo la extremidad superior derecha; el dolor empeora con el ejercicio. Al examen se observa crecimiento de las venas cutneas sobre el trax anterior. Cul de los siguientes es el diagnstico ms probable? A. B. C. D. E. Trombosis de la Vena Axilar Subclavia Insuficiencia Valvular Venosa de la Extremidad Tromboflebitis Superficial de la Mama Tromboflebitis Superficial de la Vena Ceflica Trombosis de Varicosidades Preexistentes de la Extremidad Superior. Diagnstico Trombosis de la Vena Axilar Subclavia

Respuesta A

36. Un infante de 2 meses de edad aparece bien hasta hace 3 semanas atrs cuando empez a tener disnea y dificultad para la alimentacin. Se ausculta un bajo soplo holosistlico en el borde esternal izquierdo bajo. El electrocardiograma muestra hipertrofia ventricular izquierda. Cul de los siguientes es el diagnstico ms probable? A. B. C. D. E. Bronquiolitis Aguda Alergia a la Frmula Defecto Atrioseptal Hipertensin Pulmonar Defecto Septal Ventricular Diagnstico Defecto Septal Ventricular

Respuesta E

37. Un nio de 3 aos de edad es trado al mdico por fiebre, tos y dificultad respiratoria por 2 semanas. Una radiografa de trax muestra un infiltrado en lbulo medio derecho y una gran efusin pleural. La toracocentesis muestra un fluido purulento, el cultivo del lquido crece Bacteroides melaninogenicus. La infeccin es ms probable una complicacin de cul de los siguientes? A. B. C. D. E. Fibrosis Qustica Aspiracin de Cuerpo Extrao Defecto Inmunolgico Inhalacin de un Hidrocarbono Txico Apendicitis Subaguda Diagnstico Aspiracin de Cuerpo Extrao

Respuesta B

Cual de los siguientes esta ms bajo en la sangre arterial umbilical de un feto sano que en la sangre arterial de la madre? abcdeHematocrito Afinidad del oxigeno Capacidad del oxgeno P02 Viscosidad d

Respuesta:

Tanto un paciente X como uno Y tienen frecuencia respiratoria de 12/min. Y un volumen tidal de 500 ml. Sin embargo, el paciente X tiene una PC02 arterial de 35 mm Hg, mientras el paciente Y tiene una PC02 arterial de 45 mm Hg. La diferencia en PC02 entre estos pacientes esta mejor explicada por una diferencia en cual de los siguientes? a- Gasto Cardiaco b- Espacio muerto ventilatorio c- FEV1 d- Capacidad residual funcional e- Expansin pulmonar Respuesta: b

Una paciente con un aneurisma del arco artico desarrolla ronquera. Parlisis de cual de los siguientes msculos sobre el lado izquierdo es ms probable? a- Banda anterior del digstrico b- Cricotiroideo c- Omohioideo d- Banda posterior del digstrico e- Cricoaritenoideo posterior Respuesta: e Si la arteria axilar es ligada proximalmente a su paso sobre la primera costilla, la sangre puede ser suplida a la arteria axilar distal a travs de cual de las siguientes arterias? a- Braquial b- Torcica interna c- Subclavia d- Subescapular e- Vertebral Respuesta: d

Cual de las siguientes figuras (A-E) ilustra mejor le relacin entre flujo sanguneo (blood flow) y presin arterial media (MAP) en un lecho vascular que demuestra auto regulacin del flujo sanguneo?

Respuesta: d

Se realiza una cateterizacin cardiaca en una persona sana. La muestra sangunea extrada desde un catter muestra 60% de saturacin de oxgeno y la presin obtenida muestra oscilacin de un mximo de 26 mm Hg a un mnimo de 14 mm Hg. La punta del catter fue localizada en cual de las siguientes reas? a- Ductus arterioso b- Foramen ovale c- Atrio izquierdo

d- Arteria pulmonar e- Atrio derecho Respuesta: d El procedimiento para medir el gasto cardiaco usando el principio de Fick involucra medir el consumo de oxgeno por el pulmn y medir la diferencia de la tensin de oxgeno arterio-venosa. Por la diferencia existente en el contenido de oxgeno de sangre emergiendo desde diferentes rganos, cual de los siguientes es la mejor procedencia de sangre venosa para esta medicin? a- Vena Yugular b- Arteria pulmonar c- Vena pulmonar d- Vena Safena e- Vena cava superior Respuesta: b

Luego del ayuno, un paciente no exhibe secrecin gstrica cuando mastica ni presenta relajacin proximal del estmago cuando deglute. Cul de las siguientes explica mejor estos hallazgos. a. b. c. d. e. ausencia de clulas G ausencia de clulas principales concentracin aumentada de secretina concentracin aumentada de somatostatina estmago denervado Respuesta: e

Varn de 75 aos de edad con historia de constipacin y dolores abdominales episdicos, padece de diverticulosis. Esta paciente exhibe un riesgo aumentado de presentar cul de las siguientes condiciones?:

a. b. c. d. e.

angiodisplasia carcinoma de colon hemorragia gastrointestinal colitis granulomatosa malabsorcin

Respuesta: c

Nio de 6 aos de edad presenta al examen fsico una masa suprapbica en la lnea media. Durante la ciruga se encuentra una masa qustica dependiente del ombligo y que se ubica en la base de la vejiga. Cul de las siguientes condiciones es el diagnstico ms probable?: a. b. c. d. e. hidrocele divertculo de Meckel quiste de Meckel onfalocele quiste del uraco

Respuesta: e Al cabo de 10 minutos, luego de una biopsia heptica, un paciente exhibe un dolor severo en el hombro derecho. Cul de las siguientes vas nerviosas est involucrada en la fisiopatologa del dolor que tiene el paciente?: a. b. c. d. e. axilar intercostal frnico derecho cadena simptica derecha vago derecho

Respuesta: c

Una mujer de 25 aos, presenta una historia de tres das de vmitos y evacuaciones lquidas. Presenta al examen fsico tensin arterial baja y pobre turgor en tegumentos. El sodio srico es de 130 mEq/L. Cul de los siguientes hallazgos es el ms probable?: a. b. c. d. e. concentracin srica disminuida de aldosterona incremento en las concentraciones sricas del pptido natriurtico atrial incremento en el volumen circulante efectivo incremento en la concentracin srica de vasopresina orina con osmolalidad inferior a la srica.

Respuesta: d Un hombre de 70 kg con in ingreso diario fijo de 200 mmol/da, se le est inyectando diariamente un potente mineralo corticoide por espacio de cuatro das. El tiene una ingesta de agua apropiada, a discrecin del paciente y no presenta restriccin diettica alguna. La excrecin de sodio es como sigue: Da 01 02 03 04 NaCl (mmol) 30 90 180 200

si asumimos que un litro de NaCl al 0.9 % contiene 150 mmol de NaCl y pesa 1 kg, cunto pesar este paciente al finalizar el cuarto da?: a. 66 b. 68 c. 70 d. 72 e. 74

Respuesta: d Un recin nacido masculino (atendiendo a la impronta gentica) ha desarrollado completamente los conductos sexuales masculinos y trompas de Falopio. Cul de los siguientes procesos se vio probablemente alterado durante el perodo embrionario?: a. b. c. d. e. produccin de estrgeno por los testculos produccin del factor inhibidor de Muller por los testculos produccin testicular de testosterona la respuesta de los conductos paramesonfricos a los estrgenos. Respuesta de los conductos paramesonfricos a la testosterona.

Respuesta: b

Muestras sricas en una mujer sana con historia de ciclos menstruales de 28 das muestra un pico srico en la concentracin de 17 estradiol en las pasadas 12 horas. No hay progesterona srica detectable. Dentro de los siguientes 3 das, cul de los siguientes eventos se espera que tenga lugar?:

a. b. c. d. e.

cese de la menstruacin disminucin de la temperatura basal inicio de la menstruacin ovulacin regresin del cuerpo lteo.

Respuesta: d

La lactancia no acontece durante la gestacin debido a que la accin de la prolactina es bloqueada por el incremento de cul de las siguientes hormonas?: a. b. c. d. e. estrgeno y progesterona occitocina y hormona del crecimiento prostaglandinas y hormona corinica del crecimiento insulina y cortisol tiroxina y dopamina Respuesta: a

La grfica muestra la osmolaridad orina/plsma versus el flujo urinario en pacientes humanos sanos. Cul de las siguientes situaciones puede producir un cambio de X a Y?: a. ingestin oral de un litro de agua b. ingesta oral de 200 cc de salina al 0.9 % c. ingesta oral de 200 cc de solucin de glucosa hipertnica d. ingestin oral de 200 cc de salina al 3 % e. inyeccin de ADH Respuesta: a

Un infante de 3 meses de edad presenta una masa en el cuello. Al examinar el tejido de la masa, se establece que se trata del timo. Atendiendo al origen embrionario, cul de las siguientes estructuras se encontrar en un sitio ectpico?: a. ndulo linftico jugulo digstrico b. amgdalas linguales c. glndula paratiroides d. glndula submandibular e. glndula tiroides Respuesta: c El dominio intracelular del receptor insulnico cuenta con cual de las siguientes actividades enzimticas?: a. adenilato ciclasa b. fosfodiesterasa c. fosfolipasa C d. fosfoproteinfosfatasa e. tirosina kinasa

Respuesta: e

Un adolescente de 10 aos de dad presenta quemaduras corporales severas. La prdida de nitrgeno ocurre en los primeros das luego de la quemadura. Durante este perodo agudo, cul de los siguientes sustancias juega un papel importante en la prdida del nitrgeno?: a. b. c. d. e. cortisol eritropoyetina insulina hormona paratiroidea tiroxina.

Respuesta: a

Preguntas 91-121. 91. Un jugador de bsquetbol colegial de 22 aos de edad acude al mdico por una masa escrotal izquierda; l se not por primera vez dicha masa hace 2 semanas despus de golpearse con la bola en la ingle izquierda durante un juego. El exmen abdominal no muestra anormalidades. Una masa no dolorosa de 3 cm es palpada en relacin al polo superior del testculo izquierdo; la prueba de transiluminacin es negativa. Cul de los siguientes es el diagnstico ms probable?. a. Epididimitis. b. Hematoma. c. Hernia. d. Hidrocele. e. Tumor. Respuesta correcta: E (Cncer Testicular).

92. Un joven de 17 aos de edad acude al mdico por un dolor en la ingle del lado derecho de 2 horas de evolucin; la elevacin del escroto no alivia el dolor. No hay historia de trauma. l es sexualmente activo y ha tenido mltiples parejas sexuales en los ltimos 3 aos. l no usa condn regularmente. l tuvo uretritis por Clamidia hace 1 ao tratada con Doxiciclina. El exmen muestra un escroto aumentado de volumen, edematoso, eritematoso y doloroso. El testculo izquierdo est en posicin horizontal; el reflejo cremastrico est ausente del lado derecho. Cul de los siguientes es el diagnstico ms probable?. a. Epididimitos. b. Tumor hemorrgico. c. Hernia incarcerada. d. Torsin testicular.

e. Torsin del apndice testicular. Respuesta correcta: D (Torsin testicular).

93. Un hombre de 70 aos de edad acude al mdico con una historia de 18 meses de evolucin caracterizada por un aumento en la frecuencia urinaria, vacilamiento en el inicio de la miccin y una disminucin en el calibre del chorro urinario. El examen rectal revela una prstata ligeramente aumentada de tamao y firme. Despus que l orina, se le pasa una sonda Foley recolectndose 500mL de orina residual. El urianlisis est dentro de lmites normales. Cul de los siguientes es el diagnstico ms probable?. a. Prostatitis aguda. b. Hiperplasia prosttica benigna. c. Vejiga neurgena. d. Cncer de prstata. e. Estrechez uretral. Respuesta correcta: B (Hiperplasia Prosttica Benigna).

94. Un joven de 16 aos de edad es trado al Cuarto de Urgencias con historia de 1 hora de evolucin de dolor abdominal y escrotal de inicio sbito. Hay un dolor severo a la palpacin en la regin del canal inguinal derecho; el escroto del lado derecho est vaco. El urianlis es normal. Cul de los siguientes es el manejo inicial ms efectivo?. a. Administracin de analgsicos y observacin. b. Administracin de hormonas gonadotrficas. c. Insercin de una sonda nasogstrica. d. Cistoscopa. e. Operacin inmediata. Respuesta correcta: E (Torsin Testicular con Criptorqudea).

95. Un hombre de 60 aos de edad, quien est hospitalizado, no orina desde hace 8 horas. A l se le realiz un reemplazo total de cadera hace 36 horas. l tiene una historia de diabetes mellitas tipo 1 y toma nitratos por angina de pecho estable. Durante la ciruga, la prdida de sangre estimada fue de 1500mL. Intraoperatoriamente, l tuvo un episodio corto de hipotensin que resolvi espontneamente. Su temperatura es de 37.2 (99F), tiene un pulso de 80 LPM, una FR de 16x, y una presin sangunea de 130/85 mmHg. Se observa un aumento del volumen suprapbico y una herida limpia en la cadera izquierda. El exmen fsico no muestra otras anormalidades. Su hemoglobina es de 9g/dL, los glbulos blancos estn en 11000/mm3, y el conteo de plaquetas en 150000/mm3. La glicemia capilar est en 190mg/dL. Cul de los siguientes es el paso inicial ms apropiado en el manejo de este paciente?. a. Colocacin de una sonda Foley.

b. Ultrasonido Renal. c. CAT abdominal. d. Transfusin de GRE. e. Administracin de 20 UI de insulina regular. Respuesta correcta: A (Oliguria aguda).

96. Un hombre de 25 aos de edad es trado al Cuarto de Urgencias despus de estar en los escombros debajo de un material de acero de un edificio que se derrumb. Su pulso es de 140/min, la FR es de 22x, y la presin sangunea de de 80/60 mmHg. l tiene lesiones extensas por aplastamiento de los tejidos blandos en ambos muslos sin fracturas. No hay otras lesiones obvias. Despus de la administracin IV de 3L de Lactato Ringer, su pulso es de 110/min, la FR es de 20x, y la presin sangunea es de 100/70 mmHg. Con la colocacin de una sonda Foley se obtienen 30mL de una orina vino-oscura; la prueba en orina por mioglobina es positiva. Adems de la hidratacin IV, Cul de los siguientes es el prximo paso ms apropiado en el manejo de este paciente?. a. Administracin IV de cido hidroclrico hasta obtener un pH <4. b. Administracin IV de Manitol. c. Infusin contnua de Dopamina. d. CAT con contraste oral e IV. e. Arteriografa. Respuesta correcta: B (Mioglobinuria).

97. Una mujer de 54 aos con una historia de 10 aos de evolucin de una Esclerosis Sistmica progresiva (Esclerodermia) es llevada a una laparotoma de urgencias por una apendicitis perforada con peritonitis. Durante el perodo postoperatorio inmediato, ella tena una presin sangunea de 180/110mmHg. En los siguientes 3 das, su creatinina srica se increment, y su gasto urinario disminuy a 250mL/da. Al cuarto da postoperatorio, present disnea leve. Su saturacin perifrica de Oxgeno a aire ambiente era de 89%. Los estudios en sangre mostraron un potasio en 6.2mEq/L, un BUN en 34mg/dL, y una creatinina en 3.9mg/dL. Cul de los siguientes es el prximo paso ms apropiado en el manejo de sta paciente?. a. Administracin IV de IECA. b. Administracin IV de Morfina. c. Lactato Ringer a chorro 2 L. d. Hemodilisis. e. Dilisis Peritoneal. Respuesta correcta: D (Falla Renal Aguda).

98. Una mujer de 26 aos de edad G3, P2 acude al mdico para su primer control prenatal a las 36 semanas de embarazo. Ella ha estado tomando sulfato ferroso sin receta mdica. Aunque su ingesta de protenas y caloras ha sido adecuada, ella come muy poco frutas o vegetales frescos. Esta paciente est en riesgo incrementado de la siguiente deficiencia: a. Calcio. b. cido flico. c. Vitamina A. d. Vitamina B12 (Cobalamina). e. Vitamina D. Respuesta correcta: B (Deficiencia de cido Flico).

99. Una mujer de 39 aos de edad G2, P1 tuvo un recin nacido de 3090g (6 Lb con 13 onzas) por parto vaginal espontneo. El beb tiene un puente nasal aplanado, pliegues epicnticos prominentes, y un defecto septal ventricular. Su embarazo se complic con pielonefritis a las 16 semanas de gestacin y por un resfriado a las 32 semanas. Cul de los siguientes estudios prenatales es el ms apropiado para diagnosticar la condicin de ste recin nacido?. a. Amniocentesis para cariotipo. b. Prueba de no estrs anteparto. c. Ttulos de Citomegalovirus. d. Medicin de alfa-fetoprotena srica materna. e. Ttulos de Rubola. Respuesta correcta: A (Sndrome de Down).

100. Una mujer primigesta con 42 semanas de gestacin da a luz un recin nacido de 4000 g (8Lb con 13 onzas) despus de una induccin de la labor de parto con Oxitocina. El estado 1 y 2 de la labor de parto dur 14 y 3 horas, respectivamente. Se le realiz una episiotoma media, y la placenta estaba completa. Diez minutos despus del parto, ella present un sangrado vaginal abundante estimado en 500mL en un perodo de 5 minutos. El fondo uterino se palpa suave. Cul de las siguientes es la causa ms probable de la hemorragia?. a. Laceracin cervical. b. Coagulacin intravascular diseminada. c. Tejido placentario retenido. d. Atona uterina. e. Inversin uterina. Respuesta correcta: D (Atona uterina postparto).

101. Una mujer de 32 aos de edad G5, P4 es admitida al hospital con un embarazo de 32 semanas de gestacin por presentar sangrado transvaginal rojo rutilante ligero de 2 horas de evolucin. Ella no tiene dolor ni contracciones. Sus 2 ltimos embarazos fueron cesreas. Los signos vitales son estables, y la FC fetal es de 140/min y es regular. Se toman muestras de sangre para un hemograma completo, tipaje, Rh y cruce, y se administran lquidos. Cul de los siguientes es el prximo paso ms apropiado?. a. Estudio Doppler del flujo sanguneo umbilical. b. Localizacin placentaria por exmen bimanual. c. Localizacin placentaria por ultrasonido. d. Amniocentesis para estudios de madurez pulmonar. e. Cesrea inmediata. Respuesta correcta: C (Placenta Previa).

102. Una mujer primigesta de 18 aos de edad inici labor de parto espontneamente luego de una ruptura de membranas de 14 horas de evolucin. La dilatacin cervical es de 7cm. El monitoreo de la FC fetal muestra una FC basal de 180/min con una variabilidad disminuda. Su temperatura es de 38.5 (101.3F), y su tero es doloroso a la palpacin. El frotis Gram del lquido amnitico muestra mltiples microorganismos. Cul de las siguientes es la farmacoteraputica ms apropiada?. a. Ampicilina y Gentamicina. b. Ciprofloxacina y Clindamicina. c. Eritromicina. d. Metronidazol. e. Penicilina. Respuesta correcta: A (Corioamnionitis).

103. Una mujer primigesta de 20 aos de edad con embarazo a trmino da a luz por parto vaginal y anestesia epidural un recin nacido de 3200g (7 Lb con 1 onza). Se requiri de una episiotoma media grado 2. Durante el primer da postparto, ella se queja de dolor perineal. El perin est ligeramente edematoso sin evidencia de drenaje purulento o de dehiscencia de la episiotoma. Cul de los siguientes es el prximo paso ms apropiado en el manejo de esta paciente?. a. Baos de asiento 2 veces al da. b. Injeccin local de lidocana en el perin. c. Tratamiento con Ampicilina IV. d. Morfina IM. e. Anestesia epidural. Respuesta correcta: A ( Molestas normales postepisiotoma).

104. Una mujer primigesta de 24 aos de edad con diabetes mellitas tipo 1 con un embarazo de 38 semanas de gestacin da a luz un recin nacido de 3856g (8Lb con 8 onzas). El embarazo se caracteriz por un pobre control de su diabetes. La labor de parto dur 4 horas. El Apgar fue de 7/7 al primer y tercer minuto, respectivamente. Cul de las siguientes es la prueba en sangre neonatal ms apropiada en los primeros 30 minutos luego del nacimiento?. a. Determinacin del grupo sanguneo y el factor Rh. b. Medicin del hematocrito. c. Medicin del pH. d. Medicin de la bilirrubina srica. e. Medicin de la glicemia. Respuesta correcta: E (Recin nacido de madre diabtica).

105. Un hombre de 65 aos cuadripljico como resultado de una esclerosis mltiple es hospitalizado para el tratamiento de una neumona del lbulo inferior izquierdo. Su temperatura es de 38.1 (100.5F), su pulso es de 95/min, su FR de 12x, y la presin sangunea es de 120/80 mmHg. l se ve malnutrido. Se auscultan roncus en el lbulo inferior izquierdo del pulmn. El exmen del corazn, ganglios, abdomen, y las extremidades no demostr anormalidades. Hay un rea eritematosa de 1 cm en el sacro con piel intacta y sin induracin. El exmen neurolgico muestra cuadriparesia. La sangre oculta en heces fue negativa. Cul de las siguientes es la intervencin ms efectiva para la lesin en la piel de este paciente?. a. Cambios frecuentes de posicin. b. Uso de vendajes hmedos y secos. c. Terapia de remolino. d. Terapia antibitica de amplio espectro. e. Debridamiento quirrgico. Respuesta correcta: A (lcera de decbito).

106. Desde hace 1 semana, una joven de 17 aos de edad ha tenido prurito y rash que se han incrementado. Ella no tiene historia de problemas en la piel o sntomas asociados. Ella no toma medicamentos. Su hermana con quien comparte el cuarto tuvo una condicin similar hace una semana. La temperatura de la paciente es de 36.8C (98.2F). Se observan mltiples ppulas eritematosas en el tronco de 2-5mm, especialmente en los pliegues, antebrazos, manos y dedos. No hay linfadenopatas o hepatoesplenomegalias. Cul de los siguientes es ms probablemente el organismo causal?. a. Virus Epstein-Barr. b. Estreptococo del grupo A. c. Virus del Sarampin. d. Sarcoptes scabies.

e. Virus de la varicela-zoster. Respuesta correcta: D (Escabiosis).

107. Un joven de 17 aos de edad tiene un rash que involucra la cara, el cuello, el trax superior, y las extremidades superiores e inferiores de 5 das de evolucin. Durante un viaje reciente a Mxico, l estuvo buceando encontrndose con muchos peces gelatinosos (Jellyfish). l tambin tuvo diarrea tratada con trimetoprim-sulfametoxazol. Su temperatura es de 37.8C (100F). El exmen fsico demuestra un rash concluyente, eritematoso y pruriginoso; no se observan costras ni vesculas. El exmen es por lo dems irrelevante. Cul de las siguientes es la causa ms probable de esta condicin?. a. Eczema. b. Fotosensibilidad. c. Reaccin alrgica al Jellyfish. d. Salmonella Typhi. e. Escherichia colli toxicognica. Respuesta correcta: B (Fotosensibilidad inducida por el TMP-SMX).

108. Un hombre de 48 aos de edad conocido alcohlico acude al mdico por fiebre, rash facial, y edema rpidamente progresivo de la hemicara izquierda. El edema comenz hace 12 horas luego que una costra en su barbilla izquierda le comenz a picarse. Su temperatura es de 39.2C (102.5F). l es incapaz de abrir su ojo izquierdo debido al gran edema. Cul de los siguientes es el organismo causal ms probable?. a. Estreptococo del grupo A. b. Haemophilus influenzae. c. Virus Herpes simples. d. Neisseria meningitidis. e. Streptococcus pneumoniae. Respuesta correcta: A (Infeccin por estreptococo del grupo A).

109. Un nio de 7 aos de edad es llevado al mdico 3 das despus de hacerse una laceracin pequea en la ceja izquierda. Su temperatura es de 38.8C (101.8F). El exmen fsico muestra edema y eritema de la regin periorbitaria y del prpado izquierdos con proptosis moderada y movimientos oculares disminudos y dolorosos. Los mrgenes del disco ptico son claros, y no hay anormalidades retinianas. Cul de los siguientes es el diagnstico ms probable?. a. Trombosis del Seno Cavernoso. b. Trombosis del Seno Lateral. c. Celulitis orbitaria. d. Celulitis periorbitaria (Preseptal).

e. Trombosis del Seno Sagital. Respuesta correcta: C (Celulitis Orbitaria).

110. Un hombre de 56 aos de edad presenta un rash vesicular doloroso con formacin de costras de 2 das de evolucin. l recibi quimioterapia por un Linfoma No-Hodgkin hace 1 ao. Su temperatura es de 36.7C, su pulso es de 80/min, y su presin sangunea es de 138/76mmHg. El exmen fsico por lo dems es irrelevante. Cul de los siguientes es el diagnstico ms probable?. a. Herpes zoster. b. Imptigo. c. Pioderma gangrenoso. d. Sfilis. e. Lupus eritematosos sistmico. Respuesta correcta: A (Herpes Zoster). 111. a. Dermatitis por contacto alrgica. b. Dermatitis atpica. c. Eczema dishidrtico. d. Dermatitis por contacto irritante. e. Liquen simple crnico (Neurodermatitis localizada). f. Eczema numular. g. Pitiriasis rosada. h. Psoriasis. i. Escabiasis. j. Tia corporis. Para cada paciente con una condicin de la piel pruriginosa, seleccione el diagnstico ms probable. Una mujer de 30 aos de edad acude al mdico por un rash caracterizado por costras eritematoescamosas y pruriginosas de 1 ao de evolucin; el rash es ms severo en los codos y rodillas. El rash ocurre en los meses de invierno y se alivia moderadamente con emolientes. El exmen fsico muestra placas ovales discretas de 4-6 cm de dimetro sobre las rodillas y codos; ellas tienen una base eritematosa con una escama plateada. Respuesta correcta: H (Psoriasis). Una mujer de 23 aos de edad acude al mdico porque presenta manchas plidas en su espalda desde hace 2 meses. Ella primero not la aparicin de una placa ovala de 3-4 cm en su espalda superior, seguida por la aparicin de manchas ms pequeas con escozor ocasional. Ella ha utilizado algunos medicamentos tpicos sin xito. El exmen muestra

mculas mltiples de 3-5mm en su espalda con una distribucin en arbolito de navidad; las mculas son ms plidas que la piel circundante. Respuesta correcta: G (Pitiriasis rosada).

112. Un hombre de 67 aos de edad acude al mdico por una lesin no dolorosa en la porcin superior de su frente que ha sangrado recientemente. l piensa que la lesin est all desde que el se golpe en la cabeza con una botella hace 18 meses. El exmen fsico muestra reas eritematosas en el resto de su frente. La lesin es de 1cm de dimetro con un borde definido oscuro y un centro ulcerado ms claro. Cul de los siguientes es el diagnstico ms claro?. a. Keratosis actnica. b. Epiteliona de clulas basales. c. Sarcoma de Kaposi. d. Melanoma. e. Keratosis seborreica. Respuesta correcta: B (Epitelioma de clulas basales).

113. Una mujer afroamericana de 30 aos de edad previamente sana acude al mdico por fatiga, artralgias, y un rash nodular sobre el tronco y las extremidades superiores de 3 semanas de evolucin. Se observan 12 placas nodulares induradas de 0.3-0.8cm, plidas en el trax, espalda, y en las extremidades superiores. El hgado es palpable 2 cm debajo del reborde costal derecho con una zona de matidez heptica de 14 cm, y el polo inferior del baso es palpable a 3cm por debajo del reborde costal izquierdo. No hay dolor o limitacin en la movilidad de las articulaciones. El resto del exmen muestra hallazgos normales. Un Rx de trax demuestra adenopatas hiliares bilaterales. Una biopsia de las lesiones en piel, Qu es lo ms probable que demuestre?. a. Infiltracin drmica con monolitos y clulas de Reed-Sternberg. b. Histiocitos cargados de grasa. c. Granulomas no caseosos. d. Infiltrado eosinoflico homogneo paravascular. e. Vasculitis con clulas gigantes. Respuesta correcta: C (Sarcoidosis).

114. Una mujer de 43 aos de edad que trabaja como costurera tiene una historia de 3 meses de dolor, debilidad, y adormecimiento de la mano derecha que empeora en la noche. El exmen fsico muestra hipoestesia y atrofia de la eminencia tenar. Estos hallazgos son ms probablemente causados por compresin de cul de los siguientes nervios?. a. Axilar.

b. Cutneo braquial. c. Mediano. d. Radial. e. Ulnar. Respuesta correcta: C (Sndrome del Tnel Carpal).

115. Una mujer de 50 aos de edad acude al mdico por una historia de 4 semanas de evolucin de dolor en las extremidades, dificultad en subir escaleras, y dificultad para ponerse de pie. El exmen demuestra dolor a la palpacin de los msculos cuadrceps. Hay decoloracin rojo-prpura de la piel en la frente, prpados y mejillas, as como ndulos purpreos en los codos y rodillas. Cul de los siguientes es el diagnstico ms probable?. a. Dermatomiositis. b. Enfermedad mixta del tejido conectivo. c. Psoriasis. d. Artritis Reumatoidea. e. Lupus eritematoso sistmico. Respuesta correcta: A (Dermatomiositis).

116. Una mujer de 60 aos de edad viene al mdico por dolor severo y rigidez de su cuello, hombros, y caderas de 2 meses de evolucin. Sus sntomas son ms pronunciados en la maana inmediatamente al despertarse. Ella ha tenido fatiga crnica y fiebre de bajo grado durtante este perodo. El rango del movimiento del cuello, hombros y caderas es normal. Los msculos estn mnimamente dolorosos a la palpacin. La fuerza muscular, sensacin y los reflejos tendinosos profundos son normales. La CPK es de 40 U/L y la velocidad de eritrosedimentacin est en 80mm/h. El factor reumatoide y el ANA son negativos. Cul de los siguientes es el diagnstico ms probable?. a. Fibromiositis. b. Osteoartritis. c. Polimialgia reumtica. d. Polimiositis. e. Artritis reumatoidea seronegativa. Respuesta correcta: Polimialgia Reumtica.

117. Una joven de 16 aos de edad es trada al mdico debido a dolor intermitente y edema de ambos tobillos el mes pasado. Ella actualmente no tiene dolor. Cuando el dolor ocurre, ste es tan severo que ella es incapaz de caminar. No hay fiebre ni escalofros asociados. Ella es sexualmente activa y ha tenido slo un compaero sexual en el ltimo ao. Su temperatura es de 37C (98.6F), su pulso es de 80/min, y la presin sangunea es

de 145/87mmHg. El exmen no muestra anormalidades o dolor a la palpacin de las articulaciones de los tobillos. Hay una lcera no dolorosa en la mucosa oral. Los pulmones son claros a la auscultacin. El exmen cardaco no muestra anormalidades. Los estudios de laboratorios demuestran: GB 4000/mm3. Neutrfilos segmentados 65% Eosinfilos 3% Linfocitos 25% Monocitos 7% Plaquetas 60000/mm3 VES 100mm/h. ANA 1/320 AntiDNA positivo. RPR (Reagina plasmtica rpida) 1/16. Factor reumatoide negativo. Protenas en orina 3+ Cilindros de GR negativo. GR ninguno GB 10-20/campo de alto poder. Los Rx de los tobillos no muestran anormalidades ms que edema de tejidos blandos. Cul de los siguientes es el diagnstico ms probable?. a. Enfermedad gonocccica diseminada. b. Artritis poliarticular. c. Artritis reactiva. d. Sfilis secundaria. e. Lupus eritematoso sistmico. Respuesta correcta: E (Lupus eritematoso sistmico).

118. Un hombre de 70 aos acude al mdico debido a mialgias severas y malestar general en los ltimos 2 meses. l ha perdido 2.7Kg (6Lb) durante este perodo. El exmen fsico no muestra anormalidades. Su hematocrito es del 30%, el conteo de GB es de 11300/mm3, y la VES es de 112mm/h. La CPK est dentro de lmites normales. Cul de los siguientes es el diagnstico ms probable?. a. Miopata hipertiroidea. b. Miastenia gravis. c. Polimialgia reumtica. d. Artritis reumatoide. e. Miopata seronegativa. Respuesta correcta: C (Polimialgia Reumtica).

119. Un nio de 4 aos con fractura supracondlea desplazada del hmero sin complicacin neurovascular es colocado en traccin esqueltica. Seis horas ms tarde, l tiene un dolor severo en el antebrazo que se incrementa con la extensin pasiva de la mueca y los dedos. Cul de los siguientes es el prximo paso ms apropiado en el manejo de ste nio?. a. Incrementar el peso en el aparato de traccin. b. Administracin de medicamentos analgsicos. c. Exploracin de la fractura y fasciotoma del compartimento flexor del antebrazo. d. Reduccin cerrada con el paciente bajo anestesia. e. Reduccin abierta y fijacin interna de la fractura. Respuesta correcta: C (Sndrome compartamental).

120. Una adolescente de 15 aos de edad es trada al mdico por irritabilidad, intranquilidad, dificultad para concentrarse y deterioro en la ejecucin acadmica en los ltimos 2 meses. Ella ha tenido ocasionalmente palpitaciones y ha perdido 1Kg (2Lb) de peso durante este perodo a pesar de un apetito incrementado. Menarquia ocurri a los 12 aos, y sus menstruaciones ocurren a intrvalos regulares. Ella no usa drogas y no ha iniciado relaciones sexuales. Sus padres refieren que ella no tiene problemas en la casa. El exmen no muestra anormalidades. La T3 y T4 sricas estn aumentadas. Cul de los siguientes es el diagnstico ms probable?. a. Desorden tipo dficit de atencin-hiperactividad. b. Diabetes mellitus. c. Hipertiroidismo. d. Adenoma Pituitario. e. Cncer de Tiroides. Respuesta correcta: C (Hipertiroidismo).

121. Un hombre de 49 aos de edad es llevado al Cuarto de Urgencias por una historia de 2 das de evolucin caracterizada por poliuria, polidipsia, naseas, y confusin incrementada. A l se le diagnostic un carcinoma escamoso de pulmn hace 8 semanas. l est letrgico, orientado en persona pero desorientado en tiempo y lugar. Un EKG muestra un intrvalo QT corto. Adems de la terapia con lquidos IV, cul es el manejo prximo ms apropiado en este paciente?. a. Bifosfonato IV. b. Corticoides IV. c. Manitos IV. d. Mitramicina IV. e. Hidroclorotiazida oral. Respuesta correcta: A (Hipercalcemia).

61. Un hombre de 24 aos de edad es trado al cuarto de urgencias despus de recibir una herida por cuchillo en el trax anterior izquierdo justo medial a la tetilla. El se encuentra combativo a su llegada. Su presin sangunea es 70/50 mm Hg. Las venas del cuello estn distendidas. Los sonidos respiratorios son normales bilateralmente. Cul de los siguientes es el paso ms apropiado en el manejo? (A) (B) (C) (D) (E) Rayos X de trax Intubacin endotraqueal EKG Insercin de un tubo en el lado izquierdo del trax Pericardiocentesis

Respuesta: E tem Principal Diagnstico:

hemopericardio

62. Un hombre de 60 aos de edad con enfisema es trado al cuarto de urgencias 1 hora despus de tornarse letrgico e incoherente. En los ltimos tres das, l haba incrementado los episodios de disnea, tos y produccin de esputo. Oxgeno fue administrado en la ruta hacia el hospital. A su llegada, el paciente est confuso y letrgico pero responde al estmulo del dolor. El anlisis de los gases arteriales sanguneos mientras respiraba oxgeno a 6 L/min muestra: pH PCO2 PO2 7.20 95 mm Hg 100 mm Hg

Cul de los siguientes es el paso ms apropiado en el manejo? (A) (B) (C) (D) (E) Descontinuar el oxgeno Continuar con el oxgeno actual con la adicin de un estimulante respiratorio Administracin de oxgeno a 2 L/min por puntas nasales Administracin de oxgeno al 24% por mscara de Venturi Intubacin endotraqueal y ventilacin mecnica

Respuesta: E tem Principal Diagnstico:

enfisema

63. Una mujer de 56 aos de edad est bajo evaluacin por una enfermedad obstructiva crnica recin diagnosticada. Ella ha fumado un paquete de cigarrillos diarios por 30 aos. Ella tiene una tos matutina productiva de flema y ha tenido tres a cuatro infecciones severas del tracto respiratorio superior acompaadas por respiracin entrecortada cada invierno. Ella ha tenido un estilo de vida sedentario y tiene un sobrepeso de 14 kilos (30 lb). Se escuchan sibilancias espiratorias leves. No hay cianosis o edema pedal. La espirometra muestra un FEV1 de 62% del predicho, un FVC de 73% del predicho y un radio FEV1/FVC de 65% (normal >75%). Cul de los siguientes tendra el mayor impacto sobre el pronstico a largo plazo del paciente? (A) Inmunizacin contra el virus de la influenza anualmente (B) Dejar de fumar

(C) Programa de ejercicio regular (D) Uso regular de un inhalador beta adrenrgico agonista (E) Percusin diaria y drenaje postural Respuesta: B tem Principal Diagnstico:

enfermedad pulmonar obstructiva crnica

64. Un hombre asintomtico de 54 aos de edad acude al medico para un examen de salud de mantenimiento. El nunca ha consumido alcohol. El examen muestra un aumento en la pigmentacin de la piel y angiomas en forma de araa. Existe cardiomegalia y un galope S3. El hgado est firme con un borde de 8 cm.; no hay ascitis. Los testculos son atrficos. Este paciente tiene un alto riesgo para cul de las siguientes condiciones? (A) (B) (C) (D) (E) Degeneracin cerebelar Trombosis de la vena heptica Hepatoma Insuficiencia renal Encefalopata de Wernicke

Respuesta: C tem Principal Diagnstico:

hemocromatosis

65. Diez das despus de habrsele realizado un reseccin del sigmoides y una colostoma, un hombre de 64 aos de edad tiene una temperatura de 39.7C y escalofros. l recibi gentamicina y ampicilina despus de la operacin y su curso postoperatorio haba sido sin complicaciones hasta el momento. Un cultivo de sangre fue positivo por un bacilo Gram negativo. Cul de los siguientes es el organismo causal ms probable? (A) (B) (C) (D) (E) Bacteroides fragilis Brucellla abortus Escherichia coli Proteus mirabilis Pseudomonas aeruginosa

Respuesta: A tem Principal Diagnstico:

infeccin, bacteroides fragilis

66. Para cada paciente con fatiga, selecciones el diagnstico ms probable. (A) Leucemia aguda (B) Anemia de enfermedad crnica (C) Insuficiencia cardiaca congestiva (D) Infeccin por virus de Ebstein-Barr (E) Deficiencia de cido flico (F) Deficiencia de Glucosa 6 fosfato deshidrogenada (G) Esferocitosis hereditaria (H) Hipotiroidismo (I) Deficiencia de hierro (J) Enfermedad de Lyme (K) Desorden depresivo mayor

(L) Anemia hemoltica microangioptica (M) Tuberculosis miliar (N) Deficiencia de Vitamina B12 (cobalamina) 66.1 Una mujer de 19 aos de edad acude al mdico a causa de una temperatura de 38.3 C, fatiga y dolor de garganta de una semana. El examen muestra una linfadenopata cervical y esplenomegalia. Los estudios de laboratorio iniciales muestran un conteo de leucocitos de 5000/mm3 (80% de linfocitos con muchos de stos exhibiendo caractersticas atpicas). La actividad srica de la AST (transaminasa oxaloactica) es 200 U/L. La concentracin de la bilirrubina srica y la actividad de la fosfatasa alcalina srica estn dentro de lmites normales. Respuesta: D tem Principal Diagnstico:

infeccin, virus de Ebstein-Barr

66.1 Una nia de 15 aos de edad es trada al mdico por dolorimientos, fatiga y dolor lumbar por 2 semanas. El examen mostr palidez e hipersensibilidad sobre la vrtebra y ambos fmures. Un conteo sanguneo completo mostr una concentracin de hemoglobina de 7.0 g/dL, conteo de leucocitos de 2000/mm3 y un conteo de plaquetas de 15000/mm3 Respuesta: A tem Principal Diagnstico:

leucemia aguda

67. Para cada paciente con disfagia, seleccione el diagnstico ms probable. (A) Acalasia (B) Espasmo esofgico difuso (C) Cncer esofgico (D) Candidiasis esofgica (E) Reflujo esofgico (F) Globus hystericus (G) Esofagitis herptica (H) Estrechez esofgica inferior (I) Hernia paraesofgica (J) Estrechamiento pptico del esfago (K) Divertculo faringoesofgico (Zenker) (L) Sndrome de Plummer-Vinson (M) Polimiositis (N) Parlisis pseudobulbar (O) Esclerosis sistmica (escleroderma) (P) Fstula traqueoesofgica 67.1 En los ltimos 4 meses, un hombre de 50 aos de edad ha tenido un aumento en la dificultad para digerir lquidos y slidos. l tambin ha tenido regurgitacin de los lquidos y slidos no digeridos. El examen no mostr anormalidades. Un trago de bario mostr dilatacin del esfago distal con prdida de la peristalsis en los dos tercios distales. Respuesta: A tem Principal Diagnstico:

acalasia

67.2 Por 1 mes, un hombre de 62 aos de edad haba tenido dificultad y dolor cuando tragaba; el haba tenido una prdida de peso de 4.5 Kg. (10 lb.) durante este perodo. l haba fumado dos paquetes de cigarrillos diarios por 40 aos. El examen no mostr anormalidades. Un trago de bario mostr un estrechamiento del lmen esofgico distal con obstruccin parcial. Respuesta: C tem Principal Diagnstico:

cncer esofgico

67.3 Una mujer de 68 aos de edad ha tenido dificultad para tragar solidos y lquidos en los ltimos 7 meses. Ella ha notado que sus dedos cambian de color cuando los expone al fro. El examen mostr adelgazamiento de la piel sobre las manos. El trago de bario muestra un esfago dilatado generalizadamente con prdida de la peristalsis. Respuesta: O tem Principal Diagnstico:

escleroderma

68. Una mujer de 18 aos de edad viene al cuarto de urgencias por un dolor abdominal bajo desde hace 16 horas. Ella tambin reporta prdida del apetito y nusea leve. Ella es sexualmente activa y toma contraceptivos orales. Su ltimo perodo menstrual fue 3 semanas atrs. La temperatura es 37.9 C. Existe defensa y rebote a la palpacin del cuadrante inferior derecho del abdomen; la palpacin de otra reas del abdomen resultan en dolor referido en el cuadrante inferior derecho. Los sonidos intestinales estn ausentes. El examen plvico mostr escasa descarga mucosa del orificio cervical; la palpacin del lado derecho produce dolor. No hay dolor a la movilidad cervical, y los anexos y ovarios aparentan normales. Cul de los siguientes es el diagnstico ms probable? (A) (B) (C) (D) (E) Apendicitis Embarazo ectpico Quiste ovrico Absceso tubo-ovrico Clculo ureteral

Respuesta: A tem Principal Diagnstico:

apendicitis

69. Una mujer de 40 aos de edad viene al cuarto de urgencias por fiebre y un dolor severo en aumento en el cuadrante superior derecho del abdomen por 18 horas. Ella no haba tenido nuseas o vmitos. No hay historia de intolerancia a las comidas grasas o sntomas previos. La paciente regres de un viaje a Mxico hace 6 meses. Su temperatura es 38.9C, pulso es 110/min, las respiraciones son 18/min y la presin arterial es 130/85 mm Hg. El examen muestra disminucin de los sonidos respiratorios sobre la base derecha. El examen abdominal muestra dolor a la percusin sobre el cuadrante superior derecho y sonidos intestinales normales; el signo de Murphy est ausente. La cuenta de leucocitos es 20,500/mm3 (25% bandas, 55% eosinfilos, 5% linfocitos y 15% monocitos). Cul de los siguientes es el diagnstico ms probable? (A) (B) (C) (D) (E) Absceso amebiano Apendicitis Colecistitis Hepatitis Absceso pigeno

Respuesta: A tem Principal Diagnstico:

absceso amebiano

70. Una mujer de 62 aos de edad viene al mdico porque cursa con dolor en el cuadrante inferior izquierdo del abdomen en los ltimos tres das. Ella haba tenido episodios similares en tres ocasiones previas en los ltimos 6 meses. El examen mostr dolor abdominal en el cuadrante inferior izquierdo con dolor de rebote. Su concentracin de hemoglobina es 13.6 g/L y el conteo de leucocitos es 15,400/mm3 con 82% de neutrfilos segmentados y 18% de linfocitos. El urinalisis muestra 5-6 GB/cap (campo de alto poder). Cul de los siguientes es el diagnstico ms probable? (A) (B) (C) (D) (E) Colecistitis aguda Diverticulitis aguda Pielonefritis aguda Apendicitis Peritonitis bacteriana espontnea

Respuesta: B tem Principal Diagnstico:

diverticulitis aguda

71. Un hombre de 39 aos de edad es trado al hospital por su hermano para evaluacin por confusin y olvido de las cosas en el mes pasado. Su hermano reporta que el paciente haba estado bebiendo mucho y comiendo muy poco, y haba estado ligeramente nauseoso y tembloroso. l vagaba en las noches porque no poda dormir. En la admisin al hospital la administracin de dextrosa en agua al 5% es iniciada. Dos horas despus, el paciente tiene oftalmoplega y est completamente confuso. Cul de los siguientes es el siguiente paso ms apropiado en el manejo? (A) (B) (C) (D) (E) Administracin de un anticoagulante Administracin de diazepam Administracin de grandes dosis de vitamina B1 (tiamina), intravenosamente Administracin de grandes dosis de vitamina C, intravenosamente Administracin continuada de lquidos intravenosos con magnesio

Respuesta: C tem Principal Diagnstico:

encefalopata de Wernicke

72. Un hombre de 49 aos de edad con alcoholismo viene al cuarto de urgencias porque ha tenido un dolor abdominal severamente progresivo en los ltimos dos das; el dolor ahora se irradia a la espalda. l tambin tiene nuseas y vmitos. El examen muestra defensa voluntaria en el abdomen superior con dolor a la percusin. El hematocrito es 53%, el conteo de leucocitos es 12,000/mm3 y la amilasa srica es 700 U/L. Cul de los siguientes es el siguiente paso ms apropiado en el manejo? (A) (B) (C) (D) (E) (F) Observar en el cuarto de urgencias Enviar a casa con terapia analgsica Enviar a casa con antibiticos Admitir al hospital para endoscopia Admitir al hospital para manejo mdico Admitir al hospital para una operacin E

Respuesta:

tem Principal Diagnstico:

pancreatitis aguda

73. Un hombre de 46 aos de edad viene al mdico por un dolor abdominal inferior intermitente en los pasados tres meses. No hay historia familiar de cncer. El examen no muestra otras anormalidades. Su hematocrito es 38%. La prueba de sangre oculta en heces es positiva. Los estudios de contraste del colon muestran un plipo de 1.5 cm en el colon descendente. Una serie gastrointestinal superior no muestra anormalidades. Cul de los siguientes es el siguiente paso ms apropiado en el manejo? (A) (B) (C) (D) (E) Reexaminar en un ao Repetir la prueba de sangre oculta en heces despus de 3 das de una dieta sin carnes Medir la concentracin del antgeno carcinoembrionario (CEA) srico Colonoscopa con polipectoma Colectoma total

Respuesta: D tem Principal Diagnstico:

plipo colonico

74. Una nia de 13 aos de edad anteriormente asintomtica es trada al mdico porque ha tenido dolor abdominal por 24 horas. Ella tuvo una relacin sexual por primera vez un da antes del inicio de los sntomas. El examen muestra dolor suprapbico. El urinalisis muestra de 10-25 GB/cap y 2-5 GR/cap. Los sntomas de esta paciente son ms probablemente causados por cul de los siguientes mecanismos de desplazamiento bacteriano? (A) Transmisin uretral ascendente (B) Desplazamiento ureteral descendente (C) Sembrado hematgeno (D) Contaminacin perimenarquia (E) Extensin vaginal retrgrada Respuesta: A tem Principal Diagnstico:

infeccin de vas urinarias

75. Una nia de 14 aos de edad es traida al mdico porque no ha tenido un perodo menstrual por 5 meses. La menarquia fue a los 9 aos de edad. Las menstruaciones han ocurrido siempre a intervalos crecientes. Ella tuvo relaciones sexuales con un compaero hace 5 meses. Ella est en el percentil 50 para la talla y el percentil 95 para el peso. El examen mostr hirsutismo leve con basto pelo sobre el labio superior y el rea periareolar. Hay placas liquenificadas marrn aterciopeladas e hiperpigmentadas en el cuello y los pliegues axilares. El examen plvico muestra ovarios ligeramente agrandados bilateralmente. Los estudios sricos muestran: Glucosa 250 mg/dL Hormona luteinizante 30 mUI/mL Hormona folculo estimulante 9 mUI/mL Sulfato dehidroepiandrosterona 3.7 mg/dL (N=0.8-3.4) Hormona tiroideo estimulante 1U/mL Testosterona 105 mg/dL (N=25-95) Prolactina 20 ng/mL Cul de las siguientes es la causa ms probable de la amenorrea de esta paciente? (A) Enfermedad de Addison (B) Deficiencia de 21 hidroxilasa de inicio en el adulto

(C) Sndrome de acantosis nigricans con insulino resistencia hyperandrognica (D) Teratoma ovrico (E) Embarazo Respuesta: C tem Principal Diagnstico: hyperandrognica

Sndrome

de

acantosis

nigricans

con

insulino

resistencia

76. Una mujer de 45 aos de edad viene al mdico porque tiene una historia de 3 meses con episodios en los cuales ella se siente abrumadoramente caliente y suda tanto que la ropa se torna mojada. Los episodios duran aproximadamente 5 minutos y han aumentado en frecuencia; ellos ahora ocurren al menos una vez diariamente, mayormente en la noche. Durante un episodio, su frecuencia cardiaca aument y ella se sinti ansiosa; ella tuvo una sensacin de escalofrio despus. Las menstruaciones son regulares, pero el intervalo intermenstrual ha disminuido por 5 das en los ltimos 6 meses. Ella no toma medicamentos. Ella toma cinco tazas de caf diariamente. Su temperatura es 37C, pulso es 74/min y la presin arterial es 110/70 mm Hg. El examen no muestra otras anormalidades. Cul de los siguientes es la explicacin ms probable para estos hallazgos? (A) (B) (C) (D) (E) Tumor carcinoide Ingestin excesiva de cafena Desorden de ansiedad generalizada Bochornos Hipertiroidismo

Respuesta: D tem Principal Diagnstico:

menopausia

77. Una mujer de 22 aos de edad previamente sana acude al mdico luego de 3 meses de descubrirse una masa en el cuadrante inferior externo de su mama derecha. El examen muestra una masa de 2 cm, oval, firme, lisa y mvil. No haba masas axilares presentes. La biopsia excisional mostrar ms probablemente cul de los siguientes? (A) (B) (C) (D) (E) Necrosis grasa Fibroadenoma Cambios fibroqusticos de la mama Carcinoma intraductal Papilioma intraductal

Respuesta: B tem Principal Diagnstico:

fibroadenoma

78. Para cada paciente con una masa plvica, seleccione el diagnstico ms probable (A) Quiste luteum corpus (B) Quiste dermoide (C) Embarazo ectpico (D) Endometrioma (E) Quiste folicular (F) Tumor de clulas granulosas (G) Ovario hiperestimulado

(H) tero leiomiomata (I) Tumor de clulas de Sertoli-Leydig (J) Absceso tubo-ovrico 78.1 Una mujer de 41 aos de edad acude al mdico por aumento de las menstruaciones en los ltimos 3 aos. Sus menstruaciones ocurren a intervalos regulares y ahora duran 12 das. El examen plvico muestra una masa plvica irregular y firme. Respuesta: H tem Principal Diagnstico:

tero leiomiomata

78.2 Una mujer de 26 aos de edad acude al mdico para un examen de salud de rutina. El examen plvico muestra una masa de 6 cm, qustica y anexal. Un rayos X de abdomen muestra calcificaciones. Respuesta: B tem Principal Diagnstico:

quise dermoide

78.3 Una mujer de 36 aos de edad viene al mdico para evaluacin por una historia de 4 aos de infertilidad, dismenorrea severa y aumento del dolor con las relaciones sexuales. El examen plvico muestra una masa de 5 cm en el ovario derecho. Ndulos son palpados a lo largo de los ligamentos uterosacros. Respuesta: H tem Principal Diagnstico:

Endometrioma

79. Una mujer de 34 aos de edad ha tenido hirsutismo progresivo gradualmente sobre la cara y el cuerpo en los ltimos dos aos. Ella no toma medicamentos. Su presin arterial es 116/80 mm Hg. El examen muestra una cantidad ligeramente anormal de cabello sobre el labio superior, mejillas, trax y abdomen. No hay obesidad troncal o estras moradas. El examen plvico muestra ovarios normales. El siguiente paso ms apropiado en el diagnstico es la medida de cul de las siguientes concentraciones sricas. (A) (B) (C) (D) (E) Cortisol y tiroxina (T4) Estrgeno y progesterona Hormonas folculo estimulante y luteinizante 17 -hidroxiprogesterona y prolactina Testosterona y sulfato de dehidroepiandrosterona

Respuesta: E tem Principal Diagnstico:

hirsutismo

80. Una nia de 14 aos de edad sexualmente activa es trada al cuarto de urgencias por el inicio agudo de dolor abdominal en el cuadrante superior derecho y prdida del apetito por 18 horas. Ella no tiene nuseas o vmitos. Su temperatura es 38.9C. El examen plvico bimanual muestra un exudado cervical y dolor a la movilizacin cervical. Hay dolor en el cuadrante inferior bilateral. Su cuenta de leucocitos es de 21,300/mm3. La concentracin de -hCG srica est dentro de lmites normales. Cul de los siguientes es el prximo paso ms apropiado en el manejo?

(A) (B) (C) (D) (E)

Terapia antibitica Rastreo de Meckel Culdocentesis Dilatacin y curetaje Apendectoma inmediata

Respuesta: A tem Principal Diagnstico:

enfermedad inflamatoria plvica

81. Una mujer de 20 aos de edad es trada al cuarto de urgencias por fiebre, mialgias severas, diarrea y un eritema escarlitiniforme de 4 horas. Ella se recuper de una enfermedad similar 6 meses antes. Ella est menstruando y utilizando un tampn. Aparenta estar muy enferma. Su temperatura es 40C, pulso es 130/min, y la presin arterial es 75/30. Cul de los siguientes es la farmacoterapia ms apropiada? (A) (B) (C) (D) (E) (F) Ampicilina Cloranfenicol Doxiciclina Gentamicina Nafcilina Tetraciclina

Respuesta: E tem Principal Diagnstico:

sndrome de choque txico

82. Para cada paciente con secrecin vaginal, seleccione la farmacoterapia ms apropiada (A) (B) (C) (D) (E) (F) (G) (H) (I) (J) (K) (L) Aciclovir Cefazolina Ceftriaxona Eritromicina Gentamicina Griseofulvina Metronidazol Miconazol Penicilina Espectinomicina Espiramicina Tetraciclina

82.1 Una mujer de 22 aos de edad, grava 1, para 1, acude al mdico por descarga vaginal y prurito vulvar de 7 das. Ella es alrgica a la penicilina. Su ltimo perodo menstrual fue una semana atrs. Hay una secrecin delgada, burbujeante y verde plida. Una preparacin de la secrecin en placa muestra un organismo flagelado, mvil y en forma de pera. Respuesta: G tem Principal Diagnstico:

tricomoniasis

82.2 Una paciente de 25 aos de edad, grava 2, para 2, acude al mdico por una secrecin vaginal, prurito vulvar y dispareunia de 3 das. Ella no tiene alergia a drogas. La vagina est dolorosa y edematosa. Una preparacin de hisopo con KOH muestra esporas e hifas

Respuesta: H tem Principal Diagnstico:

candidiasis

83. Una mujer de 60 aos de edad, nuligrvida viene al mdico por un sangrado vaginal ligero intermitente por 4 meses. Ella no tiene historia de enfermedad excepto por hipertensin controlada con nifedipina. La menopausia ocurri 9 aos antes. Su ltimo Papanicolau fue dos aos antes y mostr hallazgos normales. Su temperatura es 37.1C, el pulso es 84/min, la presin arterial es 138/86 mm Hg. Hay una pequea cantidad de sangre en la entrada cervical; el resto del examen es no contributorio. Cul de los siguientes es el paso ms apropiado en el manejo? (A) (B) (C) (D) (E) Reexaminar en 6 meses Terapia con estrgenos conjugados orales Colposcopa Muestreo endometrial Ablacin del endometrio

Respuesta: D tem Principal Diagnstico:

sangrado, vaginal, postmenopausia

84. Un hombre homosexual de 32 aos de edad acude al mdico por una descarga purulenta uretral desde hace 2 das. En un cultivo creci Neisseria gonorrhoeae sensible a penicilina. Una semana despus de dejar la terapia con penicilina el paciente tiene una recurrencia de la descarga uretral. Un nuevo cultivo muestra nuevamente N. gonorrhoeae sensible a penicilina. Ambos el paciente y su compaero sexual son HIV negativos. El examen del compaero sexual del paciente muestra una fisura anal; en el cultivo no creci N. gonorrhoeae. Cul de las siguientes es la causa ms probable de la recurrencia de la infeccin uretral? (A) (B) (C) (D) (E) Infeccin por herpesvirus concurrente Resistencia bacteriana emergente Ingesta excesiva de alcohol Tratamiento inadecuado con penicilina Reinfeccin por el compaero

Respuesta: E tem Principal Diagnstico:

gonorrea

85. Una mujer de 36 aos de edad previamente sana reporta hinchamiento que ha empeorado en las pasadas 4 semanas. Ella algunas veces toma uno a dos vasos de vino con las comidas pero no fuma cigarrillos. Su presin arterial es 160/95 mm Hg. El examen muestra ascitis y marcado edema de los tobillos. Unos escasos crpitos son escuchados en las bases pulmonares que rpidamente desaparecen con la inspiracin profunda. Los estudios sricos muestran: Nitrgeno de urea 30 mg/dL Creatinina 2.8 mg/dL Albmina 2.0 g/dL El urinlisis muestra 3+ de albmina y no clulas. Cul de los siguientes es la causa ms probable de la ascitis y el edema? (A) Necrosis tubular aguda

(B) (C) (D) (E)

Cirrosis del hgado Insuficiencia cardiaco congestiva Enfermedad renal glomerular Enfermedad renal intersticial

Respuesta: D tem Principal Diagnstico:

enfermedad renal glomerular

86. Una mujer de 54 aos de edad es trada al cuarto de urgencias 2 horas despus del inicio del delirio. A su llegada, ella est confusa y no cooperaba. Su temperatura es 39.4C, el pulso es 142/min y las respiraciones son regulares y a 24/min, la presin arterial es 92/50 mm Hg. El examen no muestra anormalidades. Los estudios de laboratorio muestran: Suero Glucosa Creatinina Orina Glucosa Protena Glbulos blancos ninguna 2+ 200/cap 70 mg/dL 1.4 mg/dL

En los cultivos sanguneos crecer ms probablemente cul de los siguientes? (A) (B) (C) (D) (E) (F) Bacteroides fragilis Candida albicans Escherichia coli Pseudomonas aeruginosa Staphylococcus aureus Streptococcus pyogenes (grupo A)

Respuesta: C tem Principal Diagnstico:

sepsis gram negativo

87. Un da despus de una reparacin de urgencia de un aneurisma artico roto, un paciente masculino de 66 aos de edad tiene un gasto urinario de 35 mL en un perodo de 4 horas; un catter Foley es dejado en su lugar. El recibi 14 unidades de sangre durante la operacin. Su temperatura es 37.8C, pulso es 126/min y la presin arterial es 104/68 mm Hg. El examen muestra edema perifrico difuso. Los sonidos cardacos son normales. Los pulmones son claros a la auscultacin. No hay distensin venosa yugular. El abdomen est plano. Los estudios de laboratorio muestran: Hematocrito 27% Suero Na+ 143 mEq/L K+ 5.0 mEq/L Na+ urinario 6 mEq/L Cul de los siguientes es la causa ms probable de oliguria? (A) (B) (C) (D) Insuficiencia cardiaca Hipovolemia Cateter Foley ocluido Trombosis de la arteria renal

(E) Reaccin a la transfusin Respuesta: B tem Principal Diagnstico:

oliguria

Anda mungkin juga menyukai